You are on page 1of 157

Chapter 1

Partnership Formation

PROBLEM 1: TRUE OR FALSE


1. TRUE 6. TRUE
2. TRUE 7. FALSE 1,000
3. TRUE 8. TRUE (1,000 + 1,000) x 70%
4. FALSE 700 9. TRUE
5. FALSE 10. FALSE

PROBLEM 2: FOR CLASSROOM DISCUSSION


1. A

2. D

3. Solution:
Requirement (a):
Mr. A Ms. B Totals
Cash 28,000 62,000 90,000
Accounts receivable 180,000 560,000 740,000
Inventories 114,000 193,000 307,000
Land 600,000 600,000
Building 500,000 500,000
Furniture & fixtures 50,000 35,000 85,000
Intangible assets
Total assets 972,000 1,350,000 2,322,000

Accounts payable 180,000 250,000 430,000


Other liabilities 200,000 350,000 550,000
Total liabilities 380,000 600,000 980,000

Adjusted capital balances 592,000 750,000 1,342,000

1
Requirement (b):
Cash 90,000
Accounts receivable 740,000
Inventories 307,000
Land 600,000
Building 500,000
Furniture & fixtures 85,000
Accounts payable 430,000
Other liabilities 550,000
A, Capital 592,000
B, Capital 750,000

4. Solution:
Cash 184,000
A, Capital (184,000 ÷ 2) 92,000
B, Capital (184,000 ÷ 2) 92,000

5. Solution:
Cash 184,000
A, Capital (184,000 ÷ 2) 92,000
B, Capital (184,000 ÷ 2) 92,000

The cash settlement among the partners is not recorded in the partnership’s
books because this is not a transaction of the partnership but rather a
transaction among the partners themselves.

6. Answer: None. The PFRSs permit the recognition of goodwill only when
it arises from a business combination.

2
PROBLEM 3: EXERCISES

1. Solution:
Mr. A Ms. B
Cash 20,000 30,000
Inventory 15,000
Building 40,000
Furniture & equipment 15,000
Mortgage payable (10,000)
Adjusted capital balances 35,000 75,000

2. Solutions:
Requirement (a):
Mr. Ann Ms. Buoy Totals
Cash 50,000 120,000 170,000
Accounts receivable 300,000 760,000 1,060,000
Inventories 216,000 340,000 556,000
Land 1,080,000 1,080,000
Building 900,000 900,000
Equipment 90,000 130,000 220,000
Total assets 1,736,000 2,250,000 3,986,000

Accounts payable 436,000 450,000 886,000


Mortgage payable 180,000 180,000
Total liabilities 616,000 450,000 1,066,000

Adjusted capital balances 1,120,000 1,800,000 2,920,000

Requirement (b):
Cash 170,000
Accounts receivable 1,060,000
Inventories 556,000
Land 1,080,000
Building 900,000
Equipment 220,000
Accounts payable 886,000
Mortgage payable 180,000
Ann, Capital 1,120,000
Buoy, Capital 1,800,000

3
3. Solution:
 Mr. Angot, Capital = 18,000, the sale of the land on partnership
agreement date provides information on the land’s fair value on that
date.
 M. Banglo, Capital = 40,000 cash contribution.

4. Solution:
A B C
Cash 500,000
Land 800,000
Equipment 550,000
Mortgage payable (350,000)
Adjusted capital balances 500,000 450,000 550,000

PROBLEM 4: CLASSROOM ACTIVITY

Solutions:
Requirement (a):
Partner 1 Partner 2 Totals
Cash 281,250 1,800,000 2,081,250
Accounts receivable 430,000 800,000 1,230,000
Land 1,500,000 1,500,000
Building 1,400,000 1,400,000
Total assets 3,611,250 2,600,000 6,211,250

Accounts payable 330,000 400,000 730,000


Notes payable 375,657 375,657
Provision for probable loss 300,000 300,000
Real property tax payable 40,000 40,000
Total assets 670,000 775,657 1,445,657

Adjusted capital balances 2,941,250 1,824,343 4,765,593

Requirement (b):
Cash 2,081,250
Accounts receivable 1,230,000
Land 1,500,000
Building 1,400,000
Accounts payable 730,000
Notes payable 375,657
Provision for probable loss 300,000

4
Real property tax payable 40,000
Partner 1, Capital 2,941,250
Partner 2, Capital 1,824,343
Variation #1:

Solutions:

Requirement (a) and (b):


Total net asset contributions 4,765,593
Divide by: 2
Equal credits to capital accounts 2,382,796

Partner 1 Partner 2
Equal credits to capital accounts 2,382,796 2,382,796
Fair value of net asset contribution 2,941,250 1,824,343
Bonus (558,454) 558,454

Answers: Partner 2 receives a bonus of ₱558,454.

Requirement (c): The bonus is treated as an adjustment to the equity


accounts of the partners. Partner 1’s capital shall be decreased while Partner
2’s capital shall be increased by the ₱558,454 bonus.

Requirement (d):
Cash 2,081,250
Accounts receivable 1,230,000
Land 1,500,000
Building 1,400,000
Accounts payable 730,000
Notes payable 375,657
Provision for probable loss 300,000
Real property tax payable 40,000
Partner 1, Capital 2,382,796
Partner 2, Capital 2,382,796

Variation #2:

Solutions:
Requirement (a):
Total net asset contributions 4,765,593
Divide by: 2
Equal credits to capital accounts 2,382,796

5
Partner 1 Partner 2
Equal credits to capital accounts 2,382,796 2,382,796
Fair value of net asset contribution 2,941,250 1,824,343
(Receipt) Payment (558,454) 558,454

Answer: Partner 1 shall receive cash of ₱558,454 from Partner 2.

Requirement (b):
The cash receipt and cash payment are not recorded in the partnership
books.

Requirement (c):
Cash 2,081,250
Accounts receivable 1,230,000
Land 1,500,000
Building 1,400,000
Accounts payable 730,000
Notes payable 375,657
Provision for probable loss 300,000
Real property tax payable 40,000
Partner 1, Capital 2,382,796
Partner 2, Capital 2,382,796

Variation #3:

Solutions:
Requirements (a) and (b):
Total net asset contributions 4,765,593
Divide by: 2
Equal credits to capital accounts 2,382,796

Using first Partner 1’s capital, let us determine if Partner 2’s capital
contribution has any deficiency.

Partner 1, Capital 2,941,250


Divide by: Partner 1’s equity interest 50%
Total 5,882,500
Multiply by: Partner 2's interest 50%
Minimum capital required of Partner 2 2,941,250
Partner 2's capital 1,824,343

6
Deficiency on Partner 2's capital contribution 1,116,907

Answer: Partner 2 should provide additional cash contribution of


₱1,116,907 to make his contribution proportionate to his/her interest.

Using Partner 2’s capital, let us determine if Partner 1’s capital


contribution has any deficiency.
Partner 2, Capital 1,824,343
Divide by: Partner 2’s equity interest 50%
Total 3,648,685
Multiply by: Partner 1's interest 50%
Minimum capital required of Partner 1 1,824,343
Partner 1's capital 2,941,250
Deficiency on Partner 1's capital contribution -

Conclusion: Partner 1’s contribution is not deficient.

Variation #4:

Solution:
Total net asset contributions 4,765,593
Divide by: 2
Equal credits to capital accounts 2,382,796

Partner 1 Partner 2
Equal credits to capital accounts 2,382,796 2,382,796
Fair value of net asset contribution 2,941,250 1,824,343
(Withdrawal) Additional investment (558,454) 558,454

Answer:
Partner 1 shall withdraw ₱558,454 while Partner 2 shall make an additional
investment of ₱558,454.

7
PROBLEM 5: MULTIPLE CHOICE - THEORY
1. A
2. A
3. A
4. A
5. A
6. C
7. B
8. D
9. C
10. D

PROBLEM 6: MULTIPLE CHOICE - COMPUTATIONAL


1. B (20K + 15K) = 35,000; (30K + 15K + 40K – 10K) = 75,000
2. C Algee = 50K; Beldger 80K – 35K = 45K; Ceda = 55K
3. B AAA 50K; BBB 80K-35= 45K; CCC = 55K
4. C XX 75K; YY 68K; ZZ 82.5K
5. D No goodwill (‘unidentifiable asset’) is recognized under the
bonus approach
6. A (100,000 + 200,000) = 300,000
7. C (100,000 + 200,000) x 20% = 60,000
8. B

Cash 200,000
B, capital (300,000 x 20%) 60,000
A, capital (squeeze) 140,000

9. D
Solution:
A B C Partnership
Cash 50,000 40,000 140,000 230,000
Equipment 150,000 150,000
Loan payable (40K x ½) (20,000) (20,000)
50,000 190,000 120,000 360,000
Equal interest (210 ÷ 3) 120,000 120,000 120,000 360,000
Cash receipt (payment) (70,000) 70,000 - -

10. C
Solution:
Agreed initial capital 300,000
8
A's required capital balance (300K x 25%) 75,000
B's required capital balance (300K x 75%) 225,000

A B Totals
Actual contributions 100,000 200,000 300,000
Required capital balance 75,000 225,000 300,000
Additional (Withdrawal) (25,000) 25,000 -

9
Chapter 2
Partnership Operations

PROBLEM 1: TRUE OR FALSE


6. TRUE (20 x 10%) + (20 x
1. FALSE 90% x 50%) = 11
2. TRUE 7. FALSE (20 X 50%) = 10
8. FALSE [50 + (100 – 50 –
3. TRUE 30) x 50%]
4. TRUE 9. FALSE
5. FALSE (10 – 2) x 50% = 4 10. TRUE

PROBLEM 2: FOR CLASSROOM DISCUSSION


1. C

2. C

3. D

4. D

5. D

6. Solutions:

Case #1:
A B C Total
Amount being allocated 100,000
Allocation:
1. Salaries 12,000 8,000 20,000
2. Bonus (100K - 20K) x 10% 8,000 8,000
3. Interest on cap.
(100K x 10%);(60K x 10%);(120K x 10%) 10,000 6,000 12,000 28,000
4. Allocation of remainder:
(100K - 20K - 8K - 28K) = 44K;
(44K x 40%); (44K x 30%); (44K x 30%) 17,600 13,200 13,200 44,000
As allocated 47,600 19,200 33,200 100,000

1
Case #2:
A B C Total
Amount being allocated 10,000
Allocation:
1. Salaries 12,000 8,000 20,000
2. Bonus (N/A) - -
2. Interest on cap.
(100K x 10%);(60K x 10%);(120K x 10%) 10,000 6,000 12,000 28,000
3. Allocation of remainder
(10K - 20K - 28K) = -38K
(-38K x 40%); (-38K x 30%); (-38K x 30%) (15,200) (11,400) (11,400) (38,000)
As allocated 6,800 (5,400) 8,600 10,000

Case #3:
A B C Total
Amount being allocated (20,000)
Allocation:
1. Salaries 12,000 8,000 20,000
2. Bonus (N/A) - -
2. Interest on cap.
(100K x 10%);(60K x 10%);(120K x 10%) 10,000 6,000 12,000 28,000
3. Allocation of remainder
(-20K - 20K - 28K) = -68K
(-68K x 40%); (-68K x 30%); (-68K x 30%) (27,200) (20,400) (20,400) (68,000)
As allocated (5,200) (14,400) (400) (20,000)

7. Solution:
Balance, Jan. 1, 20x1 252,000 12/12 252,000
Additional investment, July 1 72,000 6/12 36,000
Withdrawal, August 1 (27,000) 5/12 (11,250)
Weighted average capital 276,750
Multiply by: 10%
Interest 27,675

2
PROBLEM 3: EXERCISES

1. Solutions:

Case #1:
A B C Total
Amount being allocated 100,000
Allocation:
1. Bonus (10% x 100,000) 10,000 10,000
2. Interest on cap.
(80K x 6%); (50K x 6%); (30K x 6%) 4,800 3,000 1,800 9,600
3. Allocation of remainder
(100K - 10K - 9.6K) = 80.4K ÷ 3 26,800 26,800 26,800 80,400
As allocated 41,600 29,800 28,600 100,000

Case #2:
A B C Total
Amount being allocated (20,000)
Allocation:
1. Bonus (none) - -
2. Interest on cap.
(80K x 6%); (50K x 6%); (30K x 6%) 4,800 3,000 1,800 9,600
3. Allocation of remainder
(-20K - 9.6K) = -29.6K ÷ 3 (9,867) (9,867) (9,867) (29,600)
As allocated (5,067) (6,867) (8,067) (20,000)

2. Solution:
Balance, Mar. 1, 20x1 50,000 10/12 41,666.67
Additional investment, June 1 20,000 7/12 11,666.67
Withdrawal, Sept. 1 (15K - 10K) (5,000) 4/12 (1,666.67)
Weighted average capital 51,667
Multiply by: 12%
Interest on capital 6,200

3. Solutions:
Case #1:

Partner A:
Balance, Jan. 1, 20x1 120,000 12/12 120,000
Withdrawal, May 1 (20,000) 8/12 (13,333)
Additional investment, Aug. 1 10,000 5/12 4,167
Withdrawal, Oct. 1 (10,000) 3/12 (2,500)

3
Weighted Ave. Capital 108,333

Partner B:
Balance, Jan. 1, 20x1 80,000 12/12 80,000
Withdrawal, May 1 (10,000) 8/12 (6,667)
Additional investment, July 1 20,000 6/12 10,000
Withdrawal, Oct. 1 (5,000) 3/12 (1,250)
Weighted Ave. Capital 82,083

Partners Wtd. Ave. Cap.


A 108,333
B 82,083
Total 190,417

A B Total
Amount being allocated 240,000
Allocation:
(240K x 108,333/190,417);
136,543 103,457 240,000
(240K x 82,083/190,417)
As allocated 136,543 103,457 240,000

Case #2:
A B Total
Amount being allocated 240,000
Allocation:
1. Interest on cap. (see computations below) 20,000 17,000 37,000
2. Allocation of remainder
101,500 101,500 203,000
(240K - 37K) = 203K ÷ 2
As allocated 121,500 118,500 240,000

Partner A Partner B
Balance, Jan. 1, 20x1 120,000 80,000
Withdrawal, May 1 (20,000) (10,000)
Additional investment, July 1 20,000
Additional investment, Aug. 1 10,000
Withdrawal, Oct. 1 (10,000) (5,000)
Ending balances 100,000 85,000
Multiply by: 0 0
Interest on ending balance 20,000 17,000

4
4. Solutions:

Case #1:
A B Total
Amount being allocated 480,000
Allocation:
1. Salary 60,000 60,000 120,000
2. Bonus (see computation below) 60,000 60,000
3. Allocation of remainder
(480K – 120K - 60K) = 300K ÷ 2 150,000 150,000 300,000
As allocated 270,000 210,000 480,000

The bonus is computed as follows:

Profit before salaries and before bonus 480,000


Salaries (60K x 2) (120,000)
Profit after salaries but before bonus 360,000
P
B = P -
1 + Br
Where: B = bonus
P = profit before bonus and tax but after salaries
Br = bonus rate or bonus percentage

360,000
B = 360,000 -
1 + 20%
B = 360,000 - 300,000
B = 60,000

Case #2:
A B Total
Amount being allocated 480,000(a)
Allocation:
1. Salary 60,000 60,000 120,000
2. Bonus 60,000(b) 60,000
3. Allocation of remainder
(480K – 120K - 60K) = 300K ÷ 2 150,000 150,000 300,000
As allocated 270,000 210,000 360,000

(a)
Profit before salaries and bonus is computed as follows:
Profit after salaries but before bonus 360,000
Salaries (60K x 2) 120,000
Profit before salaries and bonus 480,000

5
(b)
The bonus is computed as follows:
P
B = P -
1 + Br
Where: B = bonus
P = profit before bonus and tax but after salaries
Br = bonus rate or bonus percentage

360,000
B = 360,000(c) -
1 + 20%
B = 360,000 - 300,000
B = 60,000
(c)
This is amount of profit given in the problem.

Case #3:
A B Total
Amount being allocated 480,000(a)
Allocation:
1. Salary 60,000 60,000 120,000
2. Bonus 60,000(b) 60,000
3. Allocation of remainder
(480K – 120K - 60K) = 300K ÷ 2 150,000 150,000 300,000
As allocated 270,000 210,000 360,000

(a)
Profit before salaries and bonus 480,000 (squeeze)
Salaries (60K x 2) (120,000)
Bonus (see computation below) (60,000)
Profit after salaries and bonus 300,000 (start)

(b)
The bonus is computed as follows:
 The problem states that the bonus is computed based on “Profit after
salaries and after bonus.” The “Profit after salaries and after bonus” is
actually the ₱300,000 amount given in the problem. Thus, to compute for
the bonus, the ₱300,000 amount is simply multiplied by the 20% bonus
percentage, i.e., (300,000 x 20%) = ₱60,000.

5. Answer: 0

6
PROBLEM 4: CLASSROOM ACTIVITY
The answers vary depending on the assumptions made by the students.

PROBLEM 5: MULTIPLE CHOICE - THEORY


1. A
2. D
3. D
4. A
5. A

PROBLEM 6: MULTIPLE CHOICE – COMPUTATIONAL


1. B
Solution:
Red White Total
Amount being allocated 80,000
Allocation:
1. Salaries 55,000 45,000 100,000
2. Allocation of remaining profit
(80K profit – 100K salaries) = -20K
(-20 x 60%); (-20K x 40%) (12,000) (8,000) (20,000)
As allocated 43,000 37,000 80,000

2. A
Solution:
Fox Greg Howe Total
Amount being allocated (33,000)
Allocation:
1. Salaries 30,000 - 20,000 50,000
2. Interest on capital 12,000 6,000 4,000 22,000
3. Allocation of balance
(-33K – 50K - 22K) = -105K / 3 (35,000) (35,000) (35,000) (105,000)
As allocated 7,000 (29,000) (11,000) (33,000)

3. C
Solution:
Axel Berg Cobb Total
Amount being allocated 250,000
Allocation:

7
1. Bonus to A
First 100K (100K x 10%) 10,000 10,000
Over 100K [(250K - 100K) x 20%] 30,000 30,000
2. Bonus to Berg and Cobb
(250K - 10K - 30K - 150K) x 5% 3,000 3,000 6,000
3. Allocation of bal. (204K / 3) 68,000 68,000 68,000 204,000
As allocated 108,000 71,000 71,000 250,000

4. B [140K + (40K x 6/12) – (15K x 5/12) = 153.75K x 10% = 15,375

5. B
Solution:
Let: X = profit after salaries and bonus
10%X = bonus after bonus

Choice #1 Choice #2
40,000 salary = 25,000 salary + 10%X

X is computed from the equation above:


40,000 = 25,000 + 10%X
10%X = 40,000 – 25,000
X = 15,000 / 10%
X = 150,000

Profit after salaries and bonus (X) 150,000


Multiply by: Bonus rate 10%
Bonus 15,000

Profit after salaries and bonus 150,000


Add back: Salaries (25K to Mr. A + 100K to other partners) 125,000
Add back: Bonus 15,000
Profit before salaries and bonus 290,000

6. B

7. C
Solution:
Profit (given) 46,750
Add back: Annual salary (1,000 x 12 mos.) 12,000
Add back: Interest on capital (25K x 5%) 1,250
Profit before annual salary and interest but after bonus 60,000

8
Profit before salary and interest but after bonus 60,000
Divide by: (100% less 20% bonus rate) 80%
Profit before salary, interest and bonus 75,000
Multiply by: Bonus rate 20%
Bonus (bonus before bonus scheme) 15,000

9
Chapter 3
Partnership Dissolution

PROBLEM 1: TRUE OR FALSE


1. FALSE
2. TRUE
3. TRUE
4. FALSE
5. FALSE
6. TRUE
7. FALSE (50% x 80%) = 40%
8. TRUE
9. TRUE
10. FALSE (1,000 – 100 payment) = 900

PROBLEM 2: FOR CLASSROOM DISCUSSION


1. D

2. Solutions:

Case #1:
Requirement (a):
The capital balances of the existing partners are adjusted as follows:

Carrying Fair Increase


amts. values (Decrease)
Cash 26,000 26,000 -
Accounts receivable 120,000 116,400 (3,600)
Inventory 180,000 205,000 25,000
Prepaid asset 3,600 3,600
Accounts payable (62,000) (62,000) -
Accrued liabilities (4,000) (4,000)
Net assets 264,000 285,000 21,000

Unadjusted Sh. in adjustment Adjusted


A, Capital (60%) 170,000 (21K x 60%) = 12,600 182,600
B, Capital (40%) 94,000 (21K x 40%) = 8,400 102,400
264,000 285,000

1
Date B, Capital (182,600 x 1/2) 51,200
C, Capital (182,600 x 1/2) 51,200
to record the admission of C to the partnership

Requirement (b):
Before admission Admission of C After admission
A, Capital 182,600 182,600
B, Capital 102,400 (51,200) 51,200
C, Capital - 51,200 51,200
285,000 - 285,000

Requirement (c):
Partner Before admission Admission of C After admission
A 60% 60%
B 40% -20% 20%
C 20% 20%
100% 100%

Case #2: Scenario A


Requirement (a):
The fair value of the 20% interest acquired by C is computed as follows:
Adjusted net assets before admission of C 285,000
Divide by: Interest of old partners (100% - 20%) 80%
Grossed-up fair value 356,250
Multiply by: Interest of C 20%
Fair value of C's interest 71,250

Date Cash 71,250


C, Capital 71,250
to record the admission of C to the partnership

Requirement (b):
Before admission Admission of C After admission
A, Capital 182,600 182,600
B, Capital 102,400 102,400
C, Capital 71,250 71,250
285,000 71,250 356,250

2
Requirement (c):
Partner Before admission Admission of C After admission
A 60% (100% - 20%) x 60% 48%
B 40% (100% - 20%) x 40% 32%
C 20% 20%
100% 100%

Case #2: Scenario B


Requirement (a):
Date Cash 71,250
A, Capital (100K – 71,250) x 60% 17,250
B, Capital (100K – 71,250) x 40% 11,500
C, Capital 100,000
to record the admission of C to the partnership

Requirement (b):
Before admission Admission of C After admission
A, Capital 182,600 (17,250) 165,350
B, Capital 102,400 (11,500) 90,900
C, Capital 100,000 100,000
285,000 71,250 356,250

Case #3:
Solution:
Adjusted net assets 285,000
Divide by: Existing partners' interest 3/5
Total net assets after investment by C 475,000
Multiply by: C's interest 2/5
Amt. of contribution by C 190,000

3. Solutions:
Requirement (a):
April A, Capital 320,000

3
1, B, Capital (360K – 320K) x 30%/50% 24,000
20x1
C, Capital (360K – 320K) x 20%/50% 16,000
Cash 360,000
to record the retirement of A from the
partnership

Requirement (b):
Before retirement Retirement of A After retirement
A, Capital 320,000 (320,000) -
B, Capital 192,000 (24,000) 168,000
C, Capital 128,000 (16,000) 112,000
640,000 (360,000) 280,000

Requirement (c):
Partner Before retirement Retirement of A After retirement
A 50% -50% -
B 30% 30% / (30% + 20%) 60%
C 20% 20% / (30% + 20%) 40%
100% 100%

4
PROBLEM 3: EXERCISES
1. Solutions:

Case #1:
Requirement (a):
The capital balances of the existing partners are adjusted as follows:
Carrying Fair Increase
amts. values (Decrease)
Cash 30,000 30,000 -
Accounts
140,000
receivable 120,000 (20,000)
Inventory 200,000 160,000 (40,000)
Equipment 500,000 450,000 (50,000)
Accounts payable (80,000) (80,000) -
Accrued liabilities (20,000) (20,000)
Net assets 790,000 660,000 (130,000)

Unadjusted Adjustment Adjusted


Apple, Capital (60%) 515,000 -130K x 60% = -78K 437,000
Banana, Capital (40%) 275,000 -130K x 40% = -52K 223,000
790,000 660,000

Date B, Capital (223,00 x 1/2) 111,500


C, Capital (223,00 x 1/2) 111,500
to record the admission of C to the partnership

Requirement (b):
Before admission Admission of C After admission
A, Capital 437,000 437,000
B, Capital 223,000 (111,500) 111,500
C, Capital - 111,500 111,500
660,000 - 660,000

Requirement (c):
Partner Before admission Admission of C After admission
A 60% 60%
B 40% -20% 20%
C 20% 20%
100% 100%

5
Case #2:
Requirement (a):
The fair value of the 20% interest acquired by C is computed as follows:
Adjusted net assets before admission of C 660,000
Divide by: Interest of old partners (100% - 20%) 80%
Grossed-up fair value 825,000
Multiply by: Interest of C 20%
Fair value of C's interest 165,000

Date Cash 165,000


C, Capital 165,000
to record the admission of C to the partnership

Requirement (b):
Before admission Admission of C After admission
A, Capital 437,000 437,000
B, Capital 223,000 223,000
C, Capital - 165,000 165,000
660,000 165,000 825,000

Requirement (c):
Partner Before admission Admission of C After admission
A 60% (100% - 20%) x 60% 48%
B 40% (100% - 20%) x 40% 32%
C 20% 20%
100% 100%

Case #3:
Requirement (a):
Date Cash 100,000
A, Capital (165K – 100K) x 60% 39,000
B, Capital (165K – 100K) x 40% 26,000
C, Capital 165,000
to record the admission of C to the partnership

6
Requirement (b):
Before admission Admission of C After admission
A, Capital 437,000 (39,000) 398,000
B, Capital 223,000 (26,000) 197,000
C, Capital - 165,000 165,000
660,000 100,000 760,000

Case #4:
Requirement (a):
Date Cash 165,000
C, Capital 125,000
A, Capital (165K – 125K) x 60% 24,000
B, Capital (165K – 125K) x 40% 16,000

to record the admission of C to the partnership

Requirement (b):
Before admission Admission of C After admission
A, Capital 437,000 24,000 461,000
B, Capital 223,000 16,000 239,000
C, Capital - 125,000 125,000
660,000 165,000 825,000

Case #5:
Adjusted net assets 660,000
Divide by: Existing partners' interest 3/5
Total net assets after investment by Carrots 1,100,000
Multiply by: Carrots’ interest 2/5
Amt. of contribution by Carrots 440,000

7
2. Solutions:

Case #1:
The adjusted capital balances of the partners on the date of A’s retirement
are computed as follows:

A (50%) B (30%) C (20%)


Jan. 1 320,000 192,000 128,000
Sh. In profit 400,000 240,000 160,000
Drawings (40,000) (60,000) (30,000)
Sept. 1 680,000 372,000 258,000

Requirement (a):
Sept. A, Capital 680,000
1,
B, Capital (700K – 680K) x 30%/50% 12,000
20x1
C, Capital (700K – 680K) x 20%/50% 8,000
Cash 700,000
to record the retirement of A from the
partnership

Requirement (b):
Before retirement Retirement of A After retirement
A, Capital 680,000 (680,000) -
B, Capital 372,000 (12,000) 360,000
C, Capital 258,000 (8,000) 250,000
1,310,000 (700,000) 610,000

Requirement (c):
Partner Before retirement Retirement of A After retirement
A 50% -50% -
B 30% 30% / (30% + 20%) 60%
C 20% 20% / (30% + 20%) 40%
100% 100%

8
Case #2:
Solutions:
Requirement (a):
Sept. A, Capital 680,000
1,
Cash 650,000
20x1
B, Capital (680K – 650K) x 30%/50% 18,000
C, Capital (680K – 650K) x 20%/50% 12,000

to record the retirement of A from the


partnership

Requirement (b):
Before retirement Retirement of A After retirement
A, Capital 680,000 (680,000) -
B, Capital 372,000 18,000 390,000
C, Capital 258,000 12,000 270,000
1,310,000 (650,000) 660,000

3. Solution:
A B C Total
Cash 100,000 160,000 50,000 310,000
Equipment 50,000 120,000 170,000
Capital balances - Jan. 1 150,000 160,000 170,000 480,000
Sh. In profit
(120K x 150K/480K (a));
(120K x 160K/480K);
(120K x 170K/480K) 37,500 40,000 42,500 120,000
Capital balances - Dec. 31 187,500 200,000 212,500 600,000

Since the problem does not state the partnership agreement on the sharing of
profits and losses, it is assumed that the sharing is based on the partners’
respective contributions.

4. Solutions:
Requirement (a):
The adjustments to the capital balances of A and B are computed as follows:
A B
600K x 20% [187.5K ÷ (187.5K + 200K)] (58,065)
600K x 20% [200K ÷ (187.5K + 200K)] (61,935)

9
Jan. A, Capital 58,065
1,
B, Capital 61,935
20x2
D, Capital (600,000 x 20%) 120,000
to record the admission of D to the partnership

Requirement (b):
A B C D Total
Before admission 187,500 200,000 212,500 - 600,000
Admission of D (58,065) (61,935) - 120,000 -
After admission 129,435 138,065 212,500 120,000 600,000

5. Solutions:

Requirement (a):
Dec. 31, B, Capital 200,000
20x1
Cash 164,000
A, Capital (200K – 164K) x 40%/60% 24,000
C, Capital (200K – 164K) x 20%/60% 12,000

Requirement (b):
A B C Total
Before withdrawal 187,500 200,000 212,500 600,000
Withdrawal of B 24,000 (200,000) 12,000 (164,000)
After withdrawal 211,500 - 224,500 436,000

Requirement (c):
Partner Before retirement Retirement of A After retirement
A 40% 40% / (40% + 20%) 66.67%
B 40% -40% -
C 20% 20% / (40% + 20%) 33.33%
100% 100%

10
6. Solutions:
Requirements (a) and (b):
A B Totals
Cash 11,000 22,354 33,354
Accounts receivable 214,536 532,890 747,426
Inventory 114,535 253,402 367,937
Land 603,000 603,000
Building 428,267 428,267
Equipment 50,345 34,789 85,134
Other assets - - -
Total assets 993,416 1,271,702 2,265,118
Accounts payable (178,940) (243,650) (422,590)
Notes payable (200,000) (345,000) (545,000)
Net assets 614,476 683,052 1,297,528

Requirement (c):
Adjusted net assets 1,297,528
Divide by: (100% - 20%) 80%
Grossed up fair value 1,621,910
Multiply by: C's interest 20%
Amount of need contribution 324,382

Requirement (d):
A (40%) B (40%) C (20%) Total
Fair value of net asset
contribution 614,476 683,052 324,382 1,621,910

Required capital
balance
(1,621,910 x 40%);
(1,621,910 x 40%);
(1,621,910 x 20%) 648,764 648,764 324,382 1,621,910
Cash settlement
(payment)/ receipt (34,288) 34,288 -

Requirement (e):
A (40%) B (40%) C (20%)
Adjusted capital balances, Jan. 1 648,764 648,764 324,382
Share in profit (325K x 40%);
(325K x 40%); (325K x 20%) 130,000 130,000 65,000
Drawings (50,000) (65,000) (28,000)
Capital balances, Dec. 31 728,764 713,764 361,382

11
7. Solution:
A B C Total
Before retirement 600,000 600,000 400,000 1,600,000
Revaluation of equipt.
(24K ÷ 3) 8,000 8,000 8,000 24,000
Adjusted 608,000 608,000 408,000 1,624,000
Retirement of C (408,000) (408,000)
After retirement 608,000 608,000 - 1,216,000

PROBLEM 4: CLASSROOM ACTIVITY

Case #1:

Solutions:

Income summary 50,000


A, Capital (50,000 x 40%) 20,000
B, Capital (50,000 x 60%) 30,000

Requirement (a):
B, Capital [(40,000 + 30,000) x ½] 35,000
C, Capital 35,000

Requirement (b):
A, Capital (40%) (160,000 + 20,000) 180,000
B, Capital (30%) (40,000 + 30,000 – 35,000) 35,000
C, Capital (30%) 35,000

Requirement (c):
No. It seems unfavorable because the ₱30,000 payment is lower than the
₱35,000 decrease in B’s capital account.

Case #2:

Solutions:

Income summary 50,000


A, Capital (50,000 x 40%) 20,000
B, Capital (50,000 x 60%) 30,000

Requirement (a):

A, Capital - Jan. 1 160,000

12
B, Capital - Jan. 1 40,000
Profit 50,000
Total net assets 250,000
Divide by: (100% - 20%) 80%
312,500
Multiply by: 20%
Investment by C 62,500

Cash 62,500
C, Capital 62,500

Requirement (b):
A, Capital (40% x 80% = 32%) (160,000 + 20,000) 180,000
B, Capital (60% x 80% = 48%) (40,000 + 30,000) 70,000
C, Capital ( 20%) 62,500

Case #3:

Solution:

Land 100,000
A, Capital (100,000 x 40%) 40,000
B, Capital (100,000 x 60%) 60,000

Requirement (a):

Cash 60,000
C, Capital 60,000

Requirement (b):
A, Capital (40% x 80% = 32%) (160,000 + 40,000) 200,000
B, Capital (60% x 80% = 48%) (40,000 + 60,000) 100,000
C, Capital ( 20%) 60,000

Case #4:

Solution:

Cash 50,000
C, Capital 50,000

Income summary 100,000


A, Capital (100,000 x 32%) 32,000
13
B, Capital (100,000 x 48%) 48,000
C, Capital (100,000 x 20%) 20,000

Requirement (a):
B, Capital (40,000 + 48,000) 88,000
A, Capital (32,000 x 32/52) 19,692
C, Capital (32,000 x 20/52) 12,308
Cash 120,000

Requirement (b):
A, Capital (32%/52% = 61.5%) (160,000 + 32,000 – 19,692) 172,308
C, Capital (20%/52% = 38.5%) (50,000 + 20,000 – 12,308) 57,692

PROBLEM 5: MULTIPLE CHOICE - THEORY


1. C
2. B
3. C
4. A
5. A
6. B
7. D
8. D
9. C
10. D

14
PROBLEM 6: MULTIPLE CHOICE - COMPUTATIONAL
1. B
Solution:
Total capital after admission 150,000
Multiply by: Interest of Lind 1/3
Capital credit to Lind 50,000
Contribution of Lind (40,000)
Bonus to Lind 10,000
Multiply by: Old P/L ratio of Blau 60%
Deduction to Blau's capital 6,000

Interest of Blau before admission of Lind 60,000


Deduction to Blau's capital (6,000)
Adjusted capital of Blau after admission 54,000

2. D (60K + 20K + 15K) = 95K total capital after admission x 20% =


19,000

3. A Recognition of goodwill from non-business combination


transactions is prohibited under PFRSs.

4. A
Solution:
Payment to Eddy 180,000
Capital balance of Eddy 160,000
Excess payment to Eddy 20,000

Fox Grimm
Capital balances before retirement 96,000 64,000
Share in excess payment to Eddy (12,000) (8,000)
Capital balances after retirement 84,000 56,000

5. B
Solution:
Eddy, capital 160,000
Fox, capital 96,000
Grimm, capital 64,000
Investment of Hamm 140,000
Total partnership capital after admission 460,000
Multiply by: Interest of Hamm 25%
Capital credit to Hamm 115,000

15
Investment of Hamm 140,000
Bonus to old partners (25,000)

Eddy, capital (before admission) 160,000


Share in bonus to old partners (25K x 50%) 12,500
Eddy, capital (after admission) 172,500

6. C
Solution:
Coll Maduro Prieto
(20%) (30%) (50%) Total
Unadjusted capital balance 42,000 39,000 90,000 171,000
Share in revaluation gain
[(216K – 180) x
(20%; 20% & 50%)] 7,200 7,200 21,600 36,000
Adjusted capital balance 49,200 46,200 111,600 207,000

The entry to record the settlement of Coll’s interest is as follows:


July Coll, loan 9,000
1,
Coll, Capital 49,200
20x1
Maduro, Capital (sh. in excess payment) (3K x 2/8) 750
Prieto, Capital (sh. in excess payment) (3K x 6/8) 2,250
Cash 61,200
Adjusted capital of Maduro before retirement 46,200
Share in excess payment to Coll (750)
Adjusted capital of Maduro after retirement 45,450

7. D (40K + 40K + 12K) = 92K fair value of net assets – [(5,000 x 2)


x 1 = 10,000 aggregate par value of shares issued] = 82,000
credit to share premium

8. C (1M + 300K profit – 200K payment to Partner A) = 1.1M

9. A
<List A> [(60,000 + 20,000) / 80%] x 20% = 20,000
<List B> 20,000, unaffected

10. A [50,000 + (10,000 x 4/6)] = 56,667

16
Chapter 4
Partnership Liquidation

PROBLEM 1: TRUE OR FALSE


1. FALSE
2. FALSE
3. TRUE
4. FALSE - ₱3 - ₱1 liabilities = ₱2
5. TRUE
6. TRUE
7. TRUE
8. TRUE
9. TRUE
10. TRUE
11. FALSE - ₱2 net proceeds (squeeze) - ₱5 carrying amount = (₱3 loss)
12. TRUE
13. FALSE - ₱6 - ₱1 liabilities = ₱5 available to partners x 50% = ₱2.5
14. TRUE
15. FALSE - ₱3

PROBLEM 2: FOR CLASSROOM DISCUSSION


1. D

2. D

3. Solutions:

Case #1: Lump-sum liquidation

Net cash proceeds 50,000


Carrying amount of non-cash assets (80,000)
Total loss on sale (30,000)

A (80%) B (20%) Totals


Capital balances before liquidation 20,000 18,000 38,000
Loans payable to partners 10,000 17,000 27,000
Total 30,000 35,000 65,000
Allocation of loss
(24,000) (6,000) (30,000)
(-30K x 80%); (-30K x 20%)
Amounts received by the partners 6,000 29,000 35,000

1
Checking:
Available cash (from sale) 50,000
Outside creditors (15,000)
Available cash for distribution to partners 35,000

Case #2: Installment liquidation

Net cash proceeds - first sale 45,000


Carrying amount of all non-cash assets (80,000)
Loss (35,000)

A (80%) B (20%) Totals


Capital balances before liquidation 20,000 18,000 38,000
Loans payable to partners 10,000 17,000 27,000
Total 30,000 35,000 65,000
Allocation of loss
(28,000) (7,000) (35,000)
(-35K x 80%); (-35K x 20%)
Amounts received by the partners - 1st sale 2,000 28,000 30,000

Checking:
Available cash (from 1st sale) 45,000
Outside creditors (15,000)
Available cash for distribution to partners 30,000

Case #3: Installment liquidation

Net cash proceeds - first sale 15,000


Carrying amount of all non-cash assets (80,000)
Loss (65,000)

A (80%) B (20%) Totals


Capital balances before liquidation 30,000 (28,000) 2,000
Loans payable to partners 20,000 10,000 30,000
Total 50,000 (18,000) 32,000
Allocation of loss
(52,000) (13,000) (65,000)
(-65K x 80%); (-65K x 20%)
Amounts received by the partners - 1st sale (2,000) (31,000) (33,000)

2
Answer: The partners receive nothing from the 1st sale.

Checking:
Available cash (from sale) 15,000
Outside creditors (15,000)
Available cash for distribution to partners -

4. Solutions:

Case #1: Lump-sum liquidation

Collection from accounts receivable (60K x 70%) 42,000


Sale of inventory 20,000
Sale of equipment 310,000
Liquidation costs (12,000)
Net proceeds 360,000
Carrying amt. of all non-cash assets,
except Receivable from A (60K + 120K +290K) (470,000)
Loss (110,000)

A (60%) B (40%) Totals


Capital balances before liquidation 250,000 200,000 450,000
Payable to (Receivable from) partners (10,000) 20,000 10,000
Total 240,000 220,000 460,000
Allocation of loss
(-110K x 60%); (-110K x 40%) (66,000) (44,000) (110,000)
Amounts received by the partners 174,000 176,000 350,000

Checking:
Available cash (20K on hand + 360K from sale) 380,000
Outside creditors (30,000)
Available cash for distribution to partners 350,000

Case #2: Lump-sum liquidation

Collection from accounts receivable (60K x 1/2) 30,000


Sale of inventory 20,000
Liquidation expenses (12,000)
Estimated liquidation costs (5,000)
Net proceeds 33,000
Carrying amt. of all non-cash assets, except Receivable from A (470,000)
Loss (437,000)

3
A (60%) B (40%) Totals
Capital balances before liquidation 250,000 200,000 450,000
Payable to (Receivable from) partners (10,000) 20,000 10,000
Total 240,000 220,000 460,000
Allocation of loss
(262,200) (174,800) (437,000)
(-437K x 60%); (-437K x 40%)
Total (22,200) 45,200 23,000
Allocation of deficiency to other partner 22,200 (22,200) -
Amount received by partners - 23,000 23,000

Checking:
Available cash (20K on hand + 33K from sale, net) 53,000
Outside creditors (30,000)
Available cash for distribution to partners 23,000

PROBLEM 3: EXERCISES

1. Solution:
Net cash proceeds 32,000
Carrying amount of non-cash assets (40,000)
Total loss on sale (8,000)

A (50%) B (50%) Totals


Capital balances before liquidation 20,000 15,000 35,000
Allocation of loss (-8K x 50%); (-8K x 50%) (4,000) (4,000) (8,000)
Amounts received by the partners 16,000 11,000 27,000

2. Solution:

Net cash proceeds 32,000


Carrying amount of non-cash assets (120,000)
Total loss on sale (88,000)

A (50%) B (50%) Totals


Capital balances before liquidation 60,000 45,000 105,000
Allocation of loss
(44,000) (44,000) (88,000)
(-88K x 50%); (-88K x 50%)

4
Amounts received by the partners 16,000 1,000 17,000

3. Solution:
Net proceeds 300,000
Carrying amt. of other assets (450,000)
Loss (150,000)

A (40%) B (30%) C (30%) Totals


Capital balances
60,000 270,000 45,000 375,000
before liquidation
Allocation of loss (60,000) (45,000) (45,000) (150,000)
Amounts received by
- 225,000 - 225,000
the partners

4. Solutions:

Case #1: Lump-sum liquidation

Net cash proceeds (50,000 – 5,000) 45,000


Carrying amount of non-cash assets (80,000)
Total loss on sale (35,000)

A (80%) B (20%) Totals


Capital balances before liquidation 36,000 22,000 58,000
Loans payable to partners 10,000 17,000 27,000
Total 46,000 39,000 85,000
Allocation of loss
(28,000) (7,000) (35,000)
(-35K x 80%); (-35K x 20%)
Amounts received by the partners 18,000 32,000 50,000

Checking:
Available cash (on hand + from sale, net) 20K + 45K 65,000
Outside creditors (15,000)
Available cash for distribution to partners 50,000

Case #2: Installment liquidation

Net cash proceeds - first sale (45K – 5K) 40,000


Carrying amount of all non-cash assets (80,000)
Loss (40,000)

5
A (80%) B (20%) Totals
Capital balances before liquidation 36,000 22,000 58,000
Loans payable to partners 10,000 17,000 27,000
Total 46,000 39,000 85,000
Allocation of loss
(32,000) (8,000) (40,000)
(-40K x 80%); (-40K x 20%)
Amounts received by the partners - 1st sale 14,000 31,000 45,000

Checking:
Available cash (on hand + from 1st sale, net) 20K + 40K 60,000
Outside creditors (15,000)
Available cash for distribution to partners 45,000

5. Solutions:

Case #1: Lump-sum liquidation

Collection from accounts receivable (60% x 180K) 108,000


Sale of inventory 50,000
Sale of equipment 310,000
Liquidation costs (10,000)
Net proceeds 458,000
Carrying amt. of all non-cash assets,
(650,000)
except Receivable from B (180K + 160K +310K)
Loss (192,000)

A (60%) B (40%) Totals


Capital balances before liquidation 240,000 190,000 430,000
Payable to (Receivable from) partners 20,000 (10,000) 10,000
Total 260,000 180,000 440,000
Allocation of loss
(115,200) (76,800) (192,000)
(-192K x 60%); (-192K x 40%)
Amount received by partners 144,800 103,200 248,000

Case #2: Lump-sum liquidation

Collection from accounts receivable (50% x 180K) 90,000


Sale of inventory 20,000
Sale of equipment 120,000
Liquidation expenses (10,000)

6
Estimated liquidation costs (5,000)
Net proceeds 215,000
Carrying amt. of all non-cash assets,
(650,000)
except Receivable from B (180K + 160K +310K)
Loss (435,000)

A (60%) B (40%) Totals


Capital balances before liquidation 240,000 190,000 430,000
Payable to (Receivable from) partners 20,000 (10,000) 10,000
Total 260,000 180,000 440,000
Allocation of loss
(261,000) (174,000) (435,000)
(-192K x 60%); (-192K x 40%)
Total (1,000) 6,000 5,000
Allocation of deficiency to other partner 1,000 (1,000) -
Amount received by partners - 5,000 5,000

PROBLEM 4: CLASSROOM ACTIVITY

Solutions:

Case #1: Lump-sum liquidation


The total loss on the sale is computed as follows:
Collection on accounts receivable 100,000
Sale of inventory 140,000
Sale of equipment 500,000
Liquidation expenses (4,000)
Net cash proceeds 736,000
Carrying amount of non-cash assets
(120K + 60K + 180K +600K) (960,000)
Total loss on sale (224,000)

The final settlement to partners is computed as follows:


A B
(20%) (30%) C (50%) Totals
Capital balances before
liquidation 200,000 300,000 400,000 900,000
Payable to B 40,000 40,000
Total 200,000 340,000 400,000 940,000
Allocation of loss
[224K x (20%; 30% & 50%)] (44,800) (67,200) (112,000) (224,000)

7
Amounts received by the
partners 155,200 272,800 288,000 716,000

Case #2:
The total loss on the sale is computed as follows:
Collection on account receivable 60,000
Sale of inventory 80,000
Sale of equipment 240,000
Actual liquidation expenses (4,000)
Estimated liquidation expenses (2,000)
Cash retained for future expenses (18,000)
Net cash proceeds – (net of all costs) 356,000
Carrying amount of all non-cash assets
(120K + 60K + 180K +600K) (960,000)
Total loss on sale (604,000)

The partial settlement to partners is computed as follows:


A B C Totals
Capital balances before
200,000 300,000 400,000 900,000
liquidation
Payable to B 40,000 40,000
Total 200,000 340,000 400,000 940,000
Allocation of loss
[604K x (20%; 30% &
50%)] (120,800) (181,200) (302,000) (604,000)
Amounts received by the
partners 79,200 158,800 98,000 316,000

PROBLEM 5: MULTIPLE CHOICE - THEORY


1. B
2. A
3. C
4. A
5. A

8
PROBLEM 6: MULTIPLE CHOICE – COMPUTATIONAL
1. D (348K + 232K) = 580K ÷ 80% = 725K capital after admission x
20% = 145,000

2. B
Solution:
The total loss on the sale is computed as follows:
Sale of other assets 500,000
Carrying amount of other assets (625,000)
Total loss on sale (125,000)

The partial settlement to partners is computed as follows:


Alpha Beda Totals
Capital balances before liquidation 348,000 232,000 580,000
Receivable from Beda (20,000) (20,000)
Total 348,000 212,000 560,000
Allocation of loss
[125K x (60% & 40%)] (75,000) (50,000) (125,000)
Amounts received by the partners 273,000 162,000 435,000

3. A
Solution:
The total loss on the sale is computed as follows:
Sale of other assets 385,000
Carrying amount of other assets (450,000)
Total loss on sale (65,000)
The partial settlement to partners is computed as follows:
Smith Jones Totals
Capital balances before liquidation 195,000 155,000 350,000
Receivable from Beda (20,000) - (20,000)
Total 175,000 155,000 330,000
Allocation of loss
[65K x (60% & 40%)] (39,000) (26,000) (65,000)
Amounts received by the partners 136,000 129,000 265,000

4. A
Solution:
The total loss on the sale is computed as follows:
Sale of other assets 120,000
Carrying amount of all other assets (250,000)
Total loss on sale (130,000)

9
The partial settlement to partners is computed as follows:
Cobb Davis Eddy Totals
Capital balances 80,000 90,000 70,000 240,000
Allocation of loss
[130K x (50%; 30% & 20%)] (65,000) (39,000) (26,000) (130,000)
Amounts received 15,000 51,000 44,000 110,000

5. B
Solution:

The loss is determined as follows:


A = L + E
Given information 0 = 30,000 + 570,000
Loss (squeeze) (600,000)
Adjusted balances 0 = 30,000 + (30,000)

A (30%) B (20%) C (50%) Totals


Cap. bal. - unadjusted 210,000 150,000 210,000 570,000
Allocation of loss: -600K
(180,000) (120,000) (300,000) (600,000)
x 30%; x 20%; x 50%
Total 30,000 30,000 (90,000) (30,000)
Allocation of deficiency
(-90K x 3/5); (-90K x 2/5) (54,000) (36,000) 90,000 -
Total (24,000) (6,000) - (30,000)
Additional contributions 24,000 6,000 - 30,000
Total - - - -

Allocation of loss (180,000)


Allocation of deficiency (54,000)
Decrease in A's capital balance (234,000)

6. A (Refer to solution above)

7. B (equal to carrying amount of partner’s claim)

8. A
Solution:
A = L + E
Given information 500,000 not equal to 200,000 + 490,000
Loss (squeeze) (190,000)
Adjusted balances 500,000 = 200,000 + 300,000

10
Jack (30%) Beans (70%) Totals
Capital balances – unadjusted 300,000 190,000 490,000
Allocation of loss (57,000) (133,000) (190,000)
Total 243,000 57,000 300,000

9. D
Solution:
A = L + E
Given information 120,000 not equal to - + 490,000
Loss (squeeze) (370,000)
Adjusted balances 120,000 = - + 120,000

Jack (30%) Beans (70%) Totals


Capital balances – unadjusted 300,000 190,000 490,000
Allocation of loss (111,000) (259,000) (370,000)
Total 189,000 (69,000) 120,000

10. A
Solution:
Beans (70%)
Capital balances – unadjusted 190,000
Allocation of loss (91,000) (squeeze)
Total 99,000 (start)

Total loss = (91,000) ÷ 70% = (130,000)

Jack (30%)
Capital balances – unadjusted 300,000
Allocation of loss (-130K x 30%) (39,000)
Total 261,000

11. A
Solution:
Jack (30%)
Capital balances – unadjusted 300,000
Allocation of loss (39,000) (squeeze)
Total 261,000 (start)

Total loss = (39,000) ÷ 30% = (130,000)

11
Beans (70%)
Capital balances – unadjusted 190,000
Allocation of loss (-130K x 70%) (91,000)
Total 99,000

Amount received by Jack 261,000


Amount received by Beans 99,000
Settlement of liabilities 200,000
Net proceeds from sale 560,000

12. B
Solution:
A (50%) B (25%) C (25%) Totals
Cap. bal. before liquidation 76,000 64,000 56,000 196,000
Allocation of loss (78,000) (39,000) (39,000) (156,000)
Total (2,000) 25,000 17,000 40,000
Allocation of deficiency 2,000 (1,000) (1,000) -
Total - 24,000 16,000

13. C
Solution:
Net proceeds 320,000
Carrying amount of all other assets (720,000)
Loss (400,000)

A (50%) B (30%) C (20%) Totals


Cap. bal. before liquidation 250,000 86,000 40,000 376,000
Payable to partners 64,000 20,000 84,000
Total 250,000 150,000 60,000 460,000
Allocation of loss (200,000) (120,000) (80,000) (400,000)
Total 50,000 30,000 (20,000) 520,000
Additional contribution 20,000 20,000
Total 50,000 30,000 - 540,000

14. C
Solution:
A B C
Personal assets 90,000 240,000 180,000
Personal liabilities (75,000) (150,000) (216,000)
Net free assets 15,000 90,000 (36,000)

12
15. A (100,000 x 40%) = 40,000

13
Chapter 5
Corporate Liquidation and Reorganization

PROBLEM 1: THEORY
1. D 6. D
2. D 7. E
3. A 8. B
4. D 9. A
5. D 10. C

PROBLEM 2: FOR CLASSROOM DISCUSSION

1. Solutions:

Requirement (a):

Assets pledged to fully secured creditors:


Land 1,300,000
Loan payable (750,000)
Available for unsecured creditors 550,000

Assets pledged to partially secured creditors:


Equipment - net 150,000
Notes payable (500,000)
Available for unsecured creditors -

Free assets:
Excess of land over loan payable 550,000
Cash 200,000
Accounts receivable 450,000
Total free assets 1,200,000
Unsecured liabilities with priority:
Administrative expenses (180,000)
Salaries payable (800,000)
Net free assets 220,000

1
Requirement (b):

Unsecured liabilities with priority:


Administrative expenses 180,000
Salaries payable 800,000
980,000

Fully secured creditors:


Loan payable 750,000

Partially secured creditors:


Notes payable 500,000

Unsecured liabilities without priority:


Notes payable - excess 350,000
Accounts payable 700,000
1,050,000

Requirement (c):

Total realizable value of assets 2,100,000

Less: Unsecured liabilities with priority


Salaries (800,000)
Administrative expenses (180,000) (980,000)

Less: Fully secured liabilities


Loan payable (750,000)

Less: Secured portion of partially secured


Liabilities
Notes payable (fair value of equipment) (150,000)

Excess available to unsecured liabilities without


220,000
priority (Net free assets)

Less: Unsecured liabilities without priority


Notes payable - excess over fair value of
equipment (500K - 150K) (350,000)
Accounts payable (700,000)

Estimated deficiency to unsecured non-


priority creditors (830,000)

2
Requirement (d):

Estimated recovery percentage Net free assets


of unsecured creditors without = Total unsecured liabilities
priority without priority

= 220,000 ÷ 1,050,000 (see requirement ‘b’) = 20.95%

Requirement (e):
500,000 x 20.95% = 104,761.90

Requirement (f):

BYE-BYE CORPORATION
STATEMENT OF AFFAIRS
AS OF JANUARY 1, 20X1
Available for
Book Realizable unsecured
values ASSETS values creditors
Assets pledged to fully
secured creditors:
1,000,000 Land 1,300,000
Loan payable (750,000) 550,000

Assets pledged to partially


secured creditors:
600,000 Equipment - net 150,000
Notes payable (500,000) -

Free assets:
200,000 Cash 200,000
500,000 Accounts receivable 450,000 650,000
Total free assets 1,200,000
Less: Unsecured liabilities
with priority (see below) (980,000)
Net free assets 220,000
Estimated deficiency
(squeeze) 830,000
2,300,000 Totals 1,050,000

Unsecured
Book Realizable non-priority
values LIABILITIES values liabilities
Unsecured liabilities with
priority:

3
- Administrative expenses 180,000
800,000 Salaries payable 800,000 -

Fully secured creditors:


750,000 Loan payable 750,000 -

Partially secured
creditors:
500,000 Notes payable 500,000
Equipment - net (150,000) 350,000

Unsecured creditors:
700,000 Accounts payable 700,000 700,000

(450,000) Shareholders' equity - -


2,300,000 Totals 1,050,000

2. A

3. A

4. D

5. C - Classes 1 through 6 have higher priority than Class 7.

PROBLEM 3: EXERCISES

EXERCISE 1:

Solutions:

Requirement (a):

Assets pledged to fully secured creditors:


Building - net 1,000,000
Mortgage payable (700,000)
Available for unsecured creditors 300,000

Assets pledged to partially secured creditors:


Machinery - net 300,000
Short-term bank loan (500,000)
Available for unsecured creditors -

4
Free assets:

Excess of building over mortgage payable 300,000


Cash 100,000
Accounts receivable 500,000
Inventories 500,000
Total free assets 1,400,000
Unsecured liabilities with priority:
Legal and other fees (60,000)
Income tax payable (1,000,000)
Net free assets 340,000

Requirement (b):

Unsecured liabilities with priority:


Legal and other fees 60,000
Income tax payable 1,000,000
1,060,000

Fully secured creditors:


Mortgage payable 700,000

Partially secured creditors:


Short-term bank loan 500,000

Unsecured creditors without priority


Short-term bank loan - excess 200,000
Accrued payables 300,000
Accounts payable 700,000
1,200,000

Requirement (c):

Total realizable value of assets 2,400,000

Less: Unsecured liabilities with priority


Income tax payable (1,000,000)
Legal and other fees (60,000) (1,060,000)

Less: Fully secured liabilities


Mortgage payable (700,000)

Less: Secured portion of partially secured

5
liabilities
Short-term bank loan (fair value of
(300,000)
machinery)

Excess available to unsecured liabilities


340,000
without priority (Net free assets)

Less: Unsecured liabilities without priority


Accrued payables (300,000)
Accounts payable (700,000)
Short-term bank loan - excess (500K - 300K) (200,000) (1,200,000)
Estimated deficiency to unsecured non-
priority creditors (860,000)

Requirement (d):

Estimated recovery percentage Net free assets


of unsecured creditors without = Total unsecured liabilities
priority without priority

= 340,000 ÷ 1,200,000 (see requirement ‘b’) = 28.33%

Requirement (e):
100,000 x 28.33% = 28,330

Requirement (f):
None.

Requirement (g):
GONE CORPORATION
STATEMENT OF AFFAIRS
AS OF JANUARY 1, 20X1
Available for
Realizable unsecured
Book values ASSETS values creditors
Assets pledged to fully secured
creditors:
800,000 Building - net 1,000,000
Mortgage payable (700,000) 300,000

Assets pledged to partially secured


creditors:
600,000 Machinery - net 300,000

6
Short-term bank loan (500,000) -

Free assets:
100,000 Cash 100,000
600,000 Accounts receivable 500,000
900,000 Inventories 500,000 1,100,000
Total free assets 1,400,000
Less: Unsecured liabilities with priority
(see below) (1,060,000)
Net free assets 340,000
Estimated deficiency (squeeze) 860,000
3,000,000 Totals 1,200,000

Unsecured
Realizable non-priority
Book values LIABILITIES values liabilities
Unsecured liabilities with priority:

- Legal and other fees 60,000


1,000,000 Income tax payable 1,000,000 -

Fully secured creditors:


700,000 Mortgage payable 700,000 -

Partially secured creditors:


500,000 Short-term bank loan 500,000
Machinery - net (300,000) 200,000

Unsecured creditors:
300,000 Accrued payables 300,000
700,000 Accounts payable 700,000 1,000,000

(200,000) Shareholders' equity - -


3,000,000 Totals 1,200,000

EXERCISE 2:
1. Solution:
Realizable Available for unsecured
value creditors
Assets pledged to fully
secured creditors 370,000
Fully secured creditors (260,000) 110,000

7
Free assets 320,000
Total free assets 430,000
Liabilities with priority (70,000)
Net free assets 360,000

2. Solution:
Secured and Unsecured liabilities
Priority claims without priority
Partially secured creditors 200,000
Assets pledged with partially
(120,000)
secured creditors 80,000

Unsecured creditors 540,000


Total unsecured liabilities
without priority 620,000

Net free assets 360,000


Divide by: Total unsecured liabilities without priority 620,000
Recovery percentage 58.06%

3. Solution:
Assets pledged with partially secured creditors 120,000
Partially secured creditors 200,000
Assets pledged with partially secured creditors (120,000)
Excess to be paid from net free assets 80,000
Multiply by: Recovery percentage 58.06% 46,448
Total amount paid to partially secured creditors 166,448

4. Solution:
Unsecured creditors 540,000
Multiply by: Recovery percentage 58.06%
Amount paid to unsecured creditors 313,524

8
PROBLEM 4: CLASSROOM ACTIVITY

Solutions:

Requirement (a):

Assets pledged to fully secured creditors:


Building - net 1,300,000
Notes payable (700,000)
Available for unsecured creditors 600,000

Assets pledged to partially secured creditors:


Inventories 300,000
Short-term bank loan (500,000)

Available for unsecured creditors -

Free assets:
Excess of building over loan payable 600,000
Cash 200,000
Total free assets 800,000
Unsecured liabilities with priority:
Net defined benefit liability (600,000)
Legal and other fees (100,000)
Net free assets 100,000

Requirement (b):

Unsecured liabilities with priority:


Net defined benefit liability 600,000
Legal and other fees 100,000
700,000

Fully secured creditors:


Notes payable 700,000

Partially secured creditors:


Short-term bank loan 500,000

Unsecured creditors without priority:


Short-term bank loan - excess (500K - 300K) 200,000
Accounts payable 300,000
500,000

9
Requirement (c):

Total realizable value of assets 1,800,000

Less: Unsecured liabilities with priority

Net defined benefit liability (600,000)

Legal and other fees (100,000) (700,000)

Less: Fully secured liabilities

Notes payable (700,000)

Less: Secured portion of partially secured liabilities

Short-term bank loan (fair value of inventories) (300,000)

Excess available to unsecured liabilities without


priority (Net free assets) 100,000

Less: Unsecured liabilities without priority


Short-term bank loan - excess over fair value of
inventories (500K - 300K) (200,000)

Accounts payable (300,000)

Estimated deficiency to unsecured non-priority


creditors (400,000)

Requirement (d):

Estimated recovery percentage Net free assets


of unsecured creditors without = Total unsecured liabilities
priority without priority

= 100,000 ÷ 500,000 (see requirement ‘b’) = 20%

Requirement (e):
Amount Estimated Estimated
of claim recovery % recovery
Unsecured liabilities with
priority:

10
Net defined benefit liability 600,000 100% 600,000
Legal and other fees 100,000 100% 100,000

Fully secured creditors:


Notes payable 700,000 100% 700,000

Partially secured
creditors:
Short-term bank loan (fair
100% 300,000
value of inventories) 300,000
Excess - unsecured portion 200,000 20% 40,000
Total 500,000 340,000

Unsecured creditors
without priority:
Accounts payable 300,000 20% 60,000

Shareholders' equity
Share capital 1,000,000 0% -

Total realizable value of


assets 1,800,000

Requirement (f):

FIREWOOD CORPORATION
STATEMENT OF AFFAIRS
AS OF JANUARY 1, 20X1
Available
for
Book Realizable unsecured
values ASSETS values creditors
Assets pledged to fully
secured creditors:
800,000 Building - net 1,300,000
Notes payable (700,000) 600,000

Assets pledged to partially


secured creditors:
450,000 Inventories 300,000
Short-term bank loan (500,000) -

Free assets:
200,000 Cash 200,000
100,000 Prepaid assets -
Total free assets 800,000

11
Less: Unsecured liabilities
with priority (see below) (700,000)
Net free assets 100,000
Estimated deficiency
(squeeze) 400,000
1,550,000 Totals 500,000

Unsecured
Realizable non-priority
Book values LIABILITIES values liabilities
Unsecured liabilities with
priority:
- Net defined benefit liability 600,000
600,000 Legal and other fees 100,000 -

Fully secured creditors:


700,000 Notes payable 700,000 -

Partially secured creditors:


500,000 Short-term bank loan 500,000
Inventories (300,000) 200,000

Unsecured creditors:
300,000 Accounts payable 300,000 300,000

(550,000) Shareholders' equity - -


1,550,000 Totals 500,000

PROBLEM 5: THEORY
1. B 6. D
2. C 7. B
3. A 8. D
4. D 9. A
5. C 10. D

12
PROBLEM 6: MULTIPLE CHOICE: COMPUTATIONAL
1. B
Solution:
Assets pledged to fully secured Realizable Available for
creditors: value unsecured creditors
Accounts receivable 320,000
Notes payable (280,000) 40,000

Land and building 450,000


Bank loan (250,000) 200,000

Estimated amount out of assets pledged with


fully secured creditors 240,000

2. C
Solution:
Assets pledged to fully Realizable Available for unsecured
secured creditors: value creditors
Accounts receivable 320,000
Notes payable (280,000) 40,000

Land and building 450,000


Bank loan (250,000) 200,000

Inventories 70,000
Inventories pledged to partially
(40,000)
secured creditors 30,000

Net free assets 270,000

3. B
Solution:
Available for
Realizable value
unsecured creditors
Assets pledged with fully
secured creditors 190,000
Fully secured creditors (130,000) 60,000

Free assets 140,000


Total free assets 200,000
Liabilities with priority (20,000)
Net free assets 180,000

Secured and Unsecured liabilities

13
Priority claims without priority
Partially secured creditors 100,000
Assets pledged with partially
(60,000)
secured creditors 40,000

Unsecured creditors 260,000


Total unsecured liabilities
without priority 300,000

Net free assets 180,000


Divide by: Total unsecured liabilities without
300,000
priority
Recovery percentage 60.00%

Assets pledged with partially secured creditors 60,000


Partially secured creditors 100,000
Assets pledged with partially secured creditors (60,000)
Excess to be paid from net free assets 40,000
Multiply by: Recovery percentage 60.00% 24,000
Total amount paid to partially secured creditors 84,000

4. D
Solution:
Unsecured creditors 260,000
Multiply by: Recovery percentage 60.00%
Amount paid to unsecured creditors 156,000

14
Chapter 6

Joint Arrangements

PROBLEM 1: TRUE OR FALSE


1. FALSE 6. TRUE 11. FALSE 16. TRUE
2. TRUE 7. FALSE 12. FALSE 17. TRUE
3. FALSE 8. FALSE 13. FALSE 18. TRUE
4. TRUE 9. TRUE 14. TRUE 19. FALSE
5. TRUE 10. FALSE 15. FALSE 20. FALSE

PROBLEM 2: FOR CLASSROOM DISCUSSION


1. C

2. A

3. C

4. B

5. A

6. B

7. C

8. Solutions:

Case #1: No separate books

Requirement (a):
Books of Cow Books of Chicken
a. Joint operation 300 Joint operation 300
Inventory 300 Payable to Cow 300
b. Joint operation 500 JO - Cash 300
Payable to Chicken 500 Cash 300
c. No entry Joint operation 100
JO – Cash 100
d. Receivable from Chicken 800 JO - Cash 800
Joint operation 800 Joint operation 800
e. No entry Joint operation 200
JO - Cash 200

1
Requirement (b):
Joint operation
Merchandise contributions (a) 300
Purchases (c) 100 800 Sales (d)
Expenses (e) 200 50 Unsold invty. (g)
250 Credit balance - Profit

Requirement (c):

Joint operation - Cow


Merchandise contribution (a) 300
Share in profit (250K x 50%) 125 50 Inventory taken (g)
Cash receipt (Dr. bal.) 375

Joint operation - Chicken


Cash contribution (b) 300
Share in profit (250K x 50%) 125
Cash receipt (Dr. bal.) 425

Reconciliation:
JO - Cash
Cash contribution (b) 300
Sales (d) 800 100 Additional purchases (c)
200 Expenses paid (e)
Cash balance (Dr. bal.) 800

JO - cash balance 800


Allocation:
Cash distribution to Cow 375
Cash distribution to Chicken 425
Total 800
As allocated -

2
Requirement (d):
Books of Cow Books of Chicken
g. Inventory 50 Payable to Cow 50
Joint operation 50 Joint operation 50
h. Joint operation 250 Joint operation 250
Payable to Chicken 125 Payable to Cow 125
Sh. in profit 125 Sh. in profit 125
i. Cash (squeeze) 375 Cash (squeeze) 425
Payable to Chicken 425 Payable to Cow 375
Receivable from Chicken 800 JO – cash 800

T-account analyses:

Cow’s books:
Joint operation - Cow's books
(a) 300
(b) 300 800 (d)
50 (g)
(h) 250
-

Payable to Chicken
300 (b)
(i) 425 125 (h)
-

Receivable from Chicken


(d) 800
800 (i)
-

Chicken’s books:

Joint operation - Chicken's


books
(a) 300
(c) 100 800 (d)
(e) 200 50 (g)
(h) 250
-

3
Payable to Cow
300 (a)
(g) 50 125 (h)
(i) 375
-

JO - Cash

(b) 300 100 (c)


(d) 800 200 (e)
800 (i)
-

Case #2: Separate books

Requirement (a):
Books of Cow Books of Chicken Joint operation’s
Books
a. Int. in JO 300 No entry Inventory 300
Inventory 300 Cow, capital 300
b. No entry Int. in JO 500 Cash 500
Cash Chicken, cap. 500
c. No entry No entry Inventory 100
Cash 100
d. No entry No entry Cash 800
Sales 800

COGS 350
Inventory 350
e. No entry No entry Expenses 200
Cash 200

Requirement (b):
Sales 800
Cost of sales (300 + 100 -50) (350)
Gross profit 450
Expenses (200)
Profit 250

4
Requirement (c):
Int. JO - Cow
Merchandise contribution (a) 300
Share in profit (250K x 50%) 125 50 Inventory taken (g)
Cash receipt (Dr. bal.) 375

Int. in JO - Chicken
Cash contribution (b) 300
Share in profit (250K x 50%) 125
Cash receipt (Dr. bal.) 425

Reconciliation:
Cash
Cash contribution (b) 300
Sales (d) 800 100 Additional purchases (c)
200 Expenses paid (e)
Cash balance (Dr. bal.) 800

Alternative solution:
Cow, capital
300 Merchandise contribution (a)
Inventory taken (g) 50 125 Share in profit (250K x 50%)
375 Cash receipt (Cr. bal.)

Chicken, capital
300 Cash contribution (b)
125 Share in profit (250K x 50%)
425 Cash receipt (Cr. bal.)

9. Solution:
Investment in Joint Venture
beg. 800,000
Sh. In profit Sh. In dividends
360,000 60,000
(1.2M x 30%) (200K x 30%)
1,100,000 end

10. C - Pulham Corp. shall use the equity method to account for its
investment in joint venture. Accordingly, in its financial statements (that
are not ‘separate financial statements’), Pulham shall use the ‘one-line’

5
consolidation concept. Pulham’s share in the net changes in Angels
Corp.’s net assets is accounted for in its “investment” account (balance
sheet) and “share in profit or loss of joint venture” account (statement of
comprehensive income). Therefore, the receivable is not eliminated.

PROBLEM 3: EXERCISES
1. TRUE
2. FALSE
3. TRUE
4. FALSE
5. Solution:

Joint operation - Big


Inventory
Contributions 200K 30K taken
Share in profit – (400K x 35%) 140K
Cash settlement – receipt 310K

6. Solutions:
Requirement (a):
Books of A Books of B Books of C
a. Joint operation 200 Joint operation 200 Joint operation 200
Inventory 200 Payable to A 200 Payable to A 200
b. Joint operation 10 Joint operation 10 Joint operation 10
Cash 10 Payable to A 10 Payable to A 10
c. Joint operation 400 JO – Cash 400 Joint operation 400
Payable to C 400 Payable to C 400 Cash 400
d. Joint operation 100 Joint operation 500 Joint operation 100
Payable to B 100 JO – Cash 400 Payable to B 100
Accounts
payable 100
e. Receivable JV - Cash 1,600 Receivable
from B 1,600 Joint from B 1,600
Joint operation1,600 operation 1,600 Joint operation1,600
f. Joint operation 110 Joint operation 110 Joint operation 110
Payable to B 110 Cash in bank 110 Payable to B 110

6
Books of A Books of B Books of C
h.1 Payable to C 60 Payable to C 60 Inventory 60
Joint operation 60 Joint operation 60 Joint operation 60

to charge unsold to charge unsold to record the receipt of


inventory to C inventory to C inventory from the joint
operation
h.2 Joint operation 840 Joint operation 840 Joint operation 840
Payable to B 280 Payable to A 280 Payable to A 280
Payable to C 280 Payable to C 280 Payable to B 280
Sh. in profit of JO 280 Sh. in profit of Sh. in profit of JO 280
JO 280
to record share in profits to record share in profits
and to close the JO to record share in profits and to close the JO
account in each joint and to close the JO account in each
operator’s books account in each operator’s books
operator’s books

h.3 Payable to B 490 Payable to A 490 Payable to A 490


Payable to C 620 Payable to C 620 Payable to B 490
Cash* 490 Cash* 490 Cash* 620
Receivable JO - Cash 1,600 Receivable
from B 1,600 from B 1,600

to record cash settlement to record cash to record cash


settlement settlement

Requirement (b):
Books of A Books of B Books of C
a. Interest in JO 200 No entry No entry
Inventory 200
b. Interest in JO 10 No entry No entry
Cash 10
c. No entry No entry Interest in JO 400
Cash 400
d. No entry Interest in JO 100 No entry
Accounts
payable 100
e. No entry No entry No entry
f. No entry Interest in JO 110 No entry
Cash 110

7
Separate books of the
Joint Operation
a. Inventory 200
A, Capital 200
b. Freight-in 10
A, Capital 10
c. Cash 400
C, Capital 400
d. Purchases 500
Cash 400
B, Capital 100
e. Cash 1,600
Sales 1,600
f. Expenses 110
B, Capital 110

Books of A Books of B Books of C


h.1 Interest in JO 280 Interest in JO 280 Interest in JO 280
Sh. profit of JO 280 Sh. profit of JO 280 Sh. profit of JO 280

h.2 No entry No entry Inventory 60


Interest in JO 60
h.3 Cash 490 Cash 490 Cash 620
Interest in JO 490 Interest in JO 490 Interest in JO 620
Separate books of the
Joint operation
g. Inventory, end. 60
Sales 1,600
Inventory, beg. 200
Freight-in 10
Purchases 500
Expenses 110
Income summary 840
h.1 Income summary 840
A, Capital 280
B, Capital 280
C, Capital 280
h.2 C, Capital 60
Inventory, end. 60

8
h.3 A, Capital 490
B, Capital 490
C, Capital 620
Cash 1,540

7. Solutions:

Requirement (a):
Joint operation
Merchandise – A 400 1,600 Sales – C
Purchases - A's cash 200
Merchandise – B 800 420 Unsold inventory charged to C*
Freight - in – B 40
Expenses – C 400
Profit before salary and bonus
180 - Credit balance
Salaries expense - C 60
Profit after salary but before
120 bonus - Credit balance
Bonus expense** 24
96 Profit after salary and bonus

*Unsold inventory: (P800 plus P40 freight-in) multiplied by one-half.


**Bonus is computed as follows:
P
B = P -
1 + Br
B = 120 – (120 ÷ 1.25%) = 24

Requirement (b):
Profit is allocated to the joint operators as follows:
Allocation to: A B C Totals
Profit before salary and bonus 180
Salary to C 60 (60)
Bonus to C 24 (24)
Profit after salary and bonus 96
Interest on capital:
A - (600 x 10%) 60 (60)

9
B - (840 x 10%) 84 (84)
Profit after interests on capital (48)
Allocation (24 ÷ 3) (16) (16) (16) 48
Net share - as allocated 44 68 68 -

Cash settlement is determined as follows:


Joint operation – A
Inventory contributed by A 200
Cash contribution 400
Net share in profit 44
Cash settlement – receipt 644

Joint operation - B
Inventory contributed 800
Freight paid 40
Net share in profit 68
Cash settlement – receipt 908

Joint operation –
C
Cost of
Expenses paid 400 420 inventory taken
Net share in profit 68
Cash settlement - receipt 48

10
PROBLEM 4: CLASSROOM ACTIVITY

1. Solutions:
Case #1: No separate books

Requirement (a):
Books of Tom Books of Jerry
a. Joint operation 400 Joint operation 400
Payable to Jerry 400 Inventory 400
b. Joint operation 500 JO - Cash 500
Cash 500 Payable to Tom 500
c. No entry Joint operation 200
JO – Cash 200
d. Receivable from Chicken 900 JO - Cash 900
Joint operation 900 Joint operation 900
e. No entry Joint operation 100
JO - Cash 100

Requirement (b):
Joint operation
Merchandise contributions (a) 400
Purchases (c) 200 900 Sales (d)
Expenses (e) 100 100 Unsold invty. (g)
300 Credit balance - Profit

Requirement (c):

Joint operation - Tom


Cash contribution (b) 500
Share in profit (300K x 50%) 150 200 Cash taken (h)
Cash receipt (Dr. bal.) 450

Joint operation - Jerry


Mdse. contribution (a) 400
Share in profit (300K x 50%) 150 100 Inventory taken (g)
Cash receipt (Dr. bal.) 450

Reconciliation:
JO - Cash

11
Cash contribution (b) 500
Sales (d) 900 200 Additional purchases (c)
100 Expenses paid (e)
200 Cash taken back (h)
Cash balance (Dr. bal.) 900

JO - cash balance 900


Allocation:
Cash distribution to Tom 450
Cash distribution to Jerry 450
Total 900
As allocated -

Requirement (d):

Books of Tom Books of Jerry


g. Payable to Jerry 100 Inventory 100
Joint operation 100 Joint operation 100
h. Cash 200 Payable to Tom 200
Joint operation 200 JO - Cash 200
i. Joint operation 300 Joint operation 300
Payable to Jerry 150 Payable to Tom 150
Sh. in profit 150 Sh. in profit 150
j. Cash (squeeze) 450 Cash (squeeze) 450
Payable to Jerry 450 Payable to Tom 450
Receivable from Jerry 900 JO – cash 900

Requirement (e):

Tom’s books:
Joint operation - Tom's books
(a) 400
(b) 500 900 (d)
(i) 300 100 (g)
200 (h)
-

Receivable from Jerry


(d) 900

12
900 (j)
-

Payable to Jerry
(g) 100 400 (a)
(j) 450 150 (i)
-

Jerry’s books:

Joint operation - Jerry's books


(a) 400
(c) 200 900 (d)
(e) 100 100 (g)
(i) 300
-

Payable to Tom
500 (b)
(h) 200 150 (i)
(j) 450
-

JO - Cash
(b) 500
(d) 900 200 (c)
100 (e)
200 (i)
900 (j)
-

13
Case #2: Separate books

Requirement (a):
Books of Tom Books of Jerry Joint operation’s
Books
a. No entry Int. in JO 400 Inventory 400
Inventory 400 Jerry, capital 400
b. Int. in JO 500 No entry Cash 500
Cash 500 Tom, cap. 500
c. No entry No entry Inventory 200
Cash 200
d. No entry No entry Cash 900
Sales 900

COGS 500
Inventory 500
e. No entry No entry Expenses 100
Cash 100

Requirement (b):
Sales 900
Cost of sales (400 + 200 -100) (500)
Gross profit 400
Expenses (100)
Profit 300

Requirement (c):
Int. in JO – Tom
Cash contribution (b) 500
Share in profit
150 200 Cash taken(h)
(300K x 50%)
Cash receipt (Dr. bal.) 450

Int. JO - Jerry
Merchandise contribution (a) 400
Share in profit (300K x 50%) 150 100 Inventory taken (g)
Cash receipt (Dr. bal.) 450

14
Reconciliation:
Cash
Cash contribution (b) 500
Sales (d) 900 200 Additional purchases (c)
100 Expenses paid (e)
200 Unused cash (h)
Cash balance (Dr. bal.) 900

Alternative solution:

Tom, capital
500 Cash contribution (b)
Cash withdrawal (h) 200 150 Share in profit (300K x 50%)
450 Cash receipt (Cr. bal.)

Jerry, capital
400 Merchandise contribution (a)
Inventory taken (g) 100 150 Share in profit (300K x 50%)
450 Cash receipt (Cr. bal.)

2. Solutions:
Requirement (a):
Joint operation
Mdse. contributions
120,000
(100K + 20K)
Purchases 150,000 900,000 Sales (d)
Expenses 180,000 30,000 Unsold invty. [(100K + 20K x 1/4]
480,000 Credit balance - Profit

Requirement (b):
A B C Total
Amount being allocated 480,000
Allocation:
1. Bonus (480K x 10%) 48,000 48,000
2. Allocation of remaining profit
432,000
[(480K - 48K) ÷ 3] 144,000 144,000 144,000
As allocated 192,000 144,000 144,000 480,000

15
Requirement (c):
Int. in JO - A
Mdse. Contribution 120,000
Sh. In profit 192,000 30,000 Mdse. Taken
Cash receipt (Dr. bal.) 282,000

Int. in JO – B
Cash contribution 150,000
Sh. In profit 144,000
Cash receipt (Dr. bal.) 294,000

Int. in JO – C
Cash contribution 180,000
Sh. In profit 144,000
Cash receipt (Dr. bal.) 324,000

3. Solution:

Requirement (a):
Joint operation

Purchases 1,800,000 2,700,000 Sales


Expenses 50,000
850,000 Credit balance - Profit

OR

Sales 2,700,000
Cost of sales (1M + 800K – 200K unsold) (1,600,000)
Gross profit 1,100,000
Expenses (50,000)
Loss from unsold tickets (200,000)
Profit 850,000

16
Requirement (b):

Ey Bee Total
Amount being allocated 850,000
Allocation:
1. 5% commission on purchases 50,000 40,000 90,000
2. 20% commission on sales 240,000 300,000 540,000
2. Allocation of remaining profit
220,000
(850K - 90K - 540K) / 2 110,000 110,000
As allocated 400,000 450,000 850,000

Requirement (c):
Int. in JO - Ey
Purchases 1,000,000
Expenses 20,000
Sh. In profit 400,000 1,200,000 Sales
Cash receipt (Dr. bal.) 220,000

Int. in JO – Bee
Purchases 800,000
Expenses 30,000
Sh. In profit 450,000 1,500,000 Sales
Cash payment
220,000 (Cr. bal.)

 In the final settlement, Bee pays Ey ₱220,00.

4. Solution:
Investment in Joint Venture
beg. 300,000
Sh. In profit Sh. In dividends
200,000 160,000
(500K x 40%) (400K x 40%)
340,000 end

5. Solutions:

Feb. 6 Held for Trading Securities 40,000


Commission expense 225
Cash ............................. 40,225

Mar. 31 Investment in Joint Venture 600,000


Cash ............................. 600,000

17
June 20 Cash (1,000 x ₱2.20) ............... 2,200
Dividend income ................. 2,200

June 30 Investment in Joint Venture 12,000


Share in profit of Joint Venture 12,000
(30% x 40,000)

Sept. 4 Investment in FVOCI Securities


[(4,000 x 30) + 600] ............ 120,600
Cash ............................. 120,600

Dec. 31 Held for trading securities 5,000


Unrealized gain (P/L) 5,000
(₱45 - ₱40) x 1,000

Unrealized loss (OCI) ......................... 8,600


Investment in FVOCI securities
(₱28 x 4,000) – 120,600 8,600

PROBLEM 5: MULTIPLE CHOICE - THEORY


1. B 6. B
2. A 7. A
3. C 8. D
4. D 9. A
5. C 10. A

18
PROBLEM 6: MULTIPLE CHOICE – COMPUTATIONAL
1. B
Solution:
Joint operation
Merchandise-A 8,500 20,400 Cash sales-C
Merchandise-B 7,000 4,200 Cash sales-C
Freight-in-C 200 1,210 Merchandise-B
Purchases-C 3,500 540 Unsold mdse. charged to A
Selling expenses-C 550
6,600 Profit - excess credit

2. A
Solution:
Joint operation - A
Merchandise - A 8,500
1,320 540 Unsold mdse. charged to A
9,280 Receipt - excess debit

3. C
Solution:
Joint operation
Merchandise-A 25,000
Merchandise-B 25,000
Expenses
4,450 92,650 Sales (squeeze)
(1,850 + 2,600)
38,200 Credit balances (18K + 20.2K)
4. C
Solution:
Joint operation
Merchandise-A 25,000
Merchandise-B 25,000 92,650 Sales
Expenses 4,450 2,800 Inventory taken
41,000 Profit - excess credit

5. D
Solution:
The joint operation profit is computed as follows:

Joint operation
Account with LL 16,000 18,000 Account with NN
Account with MM 32,000 42,000 Unused supplies
12,000 Profit - excess credit

19
The joint operation profit is distributed to the joint operators as follows:
LL MM NN Total
Bonus to LL 1,200 1,200
Allocation of
balance 3,600 3,600 3,600 10,800
As allocated 4,800 3,600 3,600 12,000

The net cash settlements are computed as follows:


Joint operation - LL
Balance 16,000
Sh. In profit 4,800 42,000 Inventory taken
21,200 Payment - excess credit

Joint operation – MM
Balance 32,000
Sh. In profit 3,600 - Inventory taken
Receipt - excess debit 35,600

Joint operation – NN
18,000 Balance
Sh. In profit 3,600 - Inventory taken
14,400 Payment - excess credit

From the above computations:


 LL has a net payment of 21,200.
 MM has a net receipt of 35,600.
 NN has a net payment of 14,400.

Since LL is the designated manager, he holds the joint operation’s


cash. Therefore, LL is the one who will distribute the final cash
settlement. The final settlement is as follows:
LL shall pay MM his net receipt of 35,600. In turn, LL shall
receive NN’s net payment of 14,400.

20
PROBLEM 7: MULTIPLE CHOICE – PFRS FOR SMEs
1. D

2. A

3. B

4. D

5. C

6. B

7. D
Solution:
20x1: Fair value less cost to sell (102K – 4K) = ₱98,000 lower than cost of
₱101K (cost of 100K + transaction cost of 1K).
20x2: Cost of ₱101,000 = previous carrying amount of 98K + 3K reversal of
impairment loss.
20x3: Fair value less cost to sell (90K – 4K) = ₱86,000 lower than previous
carrying amount of ₱101K.

8. E – at the year-end fair values given in the problem. Costs to sell


are ignored.

9. C

10. D

21
Chapter 7
Construction Contracts

PROBLEM 1: TRUE OR FALSE


1. FALSE 6. TRUE
2. FALSE 7. TRUE
3. FALSE 8. FALSE
4. TRUE 9. FALSE
5. TRUE 10. TRUE

PROBLEM 2: FOR CLASSROOM DISCUSSION

1. D

2. D

3. A

4. C

5. D

6. C

7. D

8. C

9. D

10. C

11. C

12. Solutions:
Requirement (a):

July 1 to Construction in progress 120,000


Dec. 31, 120,000
Cash (or other appropriate accounts)
20x1
to record the contract costs

The percentage of completion as of December 31, 20x1 is computed as


follows:

1
 The gross profit earned in 20x1 is computed as follows:

Total contract price 600,000


(a) Costs incurred to date 120,000
Estimated costs to complete 240,000
(b) Estimated total contract costs (see ‘bill of materials’) 360,000
Expected gross profit from contract 240,000
Multiply by: Percentage of completion (a) ÷ (b) 33 1/3%
Gross profit earned to date 80,000
Less: Gross profit earned in previous years -
Gross profit for the year 80,000

 The revenue and cost of construction in 20x1 are computed as


follows:

Total contract price 600,000


Multiply by: Percentage of completion 33 1/3%
Revenue to date 200,000
Less: Revenue recognized in previous yrs. -
Revenue for the year 200,000
Cost of construction (squeezed) (120,000)
Gross profit for the year (see computation above) 80,000

The year-end adjusting entry to recognize revenue is as follows:


Dec. 31, Cost of construction 120,000
20x1 Construction in progress (gross profit) 80,000
Revenue 200,000

Dec. Receivable (600K x 33 1/3%) 200,000


31, 180,000
Progress billings (given)
20x1
Contract liability 20,000

to record the billing to the customer

“Receivable” is debited instead of “Contract asset” because Contractor Co.


has an unconditional right to consideration for progress made on the
contract.

The excess of the receivable (i.e., unconditional right to consideration) over


the amount invoiced to the customer (i.e., progress billing) is recognized as a
contract liability.

Contract liability – is an entity’s obligation to transfer goods or services to a


customer for which the entity has received consideration (or the amount is
due) from the customer.

2
Dec. Cash 60,000
31,
Receivable 60,000
20x1
to record the collection on the billing

Requirement (b):
Contractor Co.
Statement of financial position
As of December 31, 20x1

Current assets
Receivable (200,000 - 60,000) 140,000
Contract asset* 20,000
Total current assets 160,000

Current liabilities
Contract liability (see journal entries above) 20,000
Total current liabilities 20,000

*Construction in progress (120,000 + 80,000) 200,000


Progress billing (180,000)
Contract asset 20,000

Contractor Co.
Statement of profit or loss
For the year ended December 31, 20x1

Revenue 200,000
Cost of construction (120,000)
Gross profit 80,000
Other operating expenses -
Profit for the year 80,000

13. Solutions:
20x1 20x2
Total contract price 9,000,000 9,000,000
(a) Costs incurred to date 3,900,000 6,300,000
Estimated costs to complete (squeeze) 3,900,000 1,800,000
(b) Estimated total contract costs 7,800,000 8,100,000
Expected profit (loss) 1,200,000 900,000
Multiply by: % of completion (a) ÷ (b) 50% 77.7778%
Profit (loss) to date 600,000 700,000
Profit recognized in prior years - (600,000)

3
Profit (loss) for the year 600,000 100,000

20x1 20x2
Total contract price 9,000,000 9,000,000
Multiply by: % of completion 50% 77.7778%
Contract revenue to date 4,500,000 7,000,000
Contract revenue in prior years - (4,500,000)
Contract revenue for the year 4,500,000 2,500,000
Cost of construction (squeeze) (3,900,000) (2,400,000)
Profit (loss) for the year 600,000 100,000

14. Solutions:
20x1 20x2
Contract revenue to date (a) 3,900,000 6,300,000
Contract revenue in prior years - (3,900,000)
Contract revenue for the year 3,900,000 2,400,000
Cost of construction (b) (3,900,000) (2,400,000)
Profit (loss) for the year - -

(a)
Equal to the “Cumulative contract costs incurred.”
(b)
Equal to the costs incurred during the year. The cost incurred in 20x2 is
computed as follows: (6,300,000 – 3,900,000) = 2,400,000.

15. Solution:
No revenue shall be recognized during the course of construction. Revenue
(and cost of construction) will be recognized only when the construction is
complete and legal title over the constructed building is transferred to the
customer.

16. Solutions:
 The costs incurred to date include the cost of an uninstalled materials
(i.e., elevators).
 Because all the conditions under PFRS 15 are met, the entity shall adjust
its measure of progress to recognize revenue only to the extent of the
costs of the uninstalled elevators. The cost of goods sold recognized
in 20x1 will also include this cost. Consequently, the entity recognizes
zero profit from the elevators in 20x1.

Percentage of Total costs incurred to date


=
completion Estimated total contract costs
= (500,000 costs incurred, excluding cost of elevator) ÷ (2.5M ‘other costs’
only, excluding cost of elevator)
Percentage of completion = 20%

Requirement (a): Revenue in 20X2

4
[(5M transaction price – 1.5M cost of elevator) x 20%] + 1.5M cost of elevator
= ₱2,200,000 revenue in 20X2

Requirement (b): Profit in 20X2


Cost of goods sold in 20X2
[(2.5M ‘other costs’ only, excluding cost of elevator) x 20%] + 1.5M cost of
elevator = ₱2,000,000 cost of goods sold in 20X2

Profit in 20X2 = 2.2M – 2M = ₱200,000

17. Solutions:

Analysis:
Since the additional goods or services to be provided in the modified contract
are not distinct, they are essentially a part of a single performance obligation
that is only partially satisfied. Therefore, the contract modification is
accounted for as if it were a part of the existing contract.

Accordingly, the effect of the contract modification on the transaction price,


and on the entity’s measure of progress towards complete satisfaction of the
performance obligation, is recognized as an increase or decrease in revenue
at the date of the contract modification. The adjustment to revenue is
made on a cumulative catch-up basis.

The percentage of completion is computed as follows:


Contract
modification date
20x1 in 20x2
(a) Costs incurred to date 420,000 420,000
Estimated costs to complete ignored ignored
(b) Estimated total contract costs (given) 700,000 820,000 (1)
Percentage of completion (a) ÷ (b) 60% 51.2%

(1)
The revised estimated total contract costs as of the date of contract
modification in 20x2 is computed as (700K original estimate of total contract
costs + 120K increase due to the contract modification in 20x2) = 820K.

The revenue in 20x1 and the cumulative catch-up adjustment to


revenue in 20x2 are computed as follows:
Contract
modification
20x1 date in 20x2
Total contract price 1,000,000 1,350,000 (2)
Multiply by: % of completion 60% 51.2%
Revenue to date 600,000 691,200
Less: Revenue recognized in prior yrs. - (600,000)

5
Revenue in 20x1 /
Cumulative catch-up adjustment to
revenue in 20x2 600,000 91,200
Cost of construction (420,000) ( - )
Gross profit for the year /
Cumulative catch-up adjustment to gross
profit in 20x2 180,000 91,200

(2)
The bonus is included in the transaction price only in 20x2 when it became
highly probable that the entity will receive the bonus. The revised
transaction price on contract modification date in 20x2 is computed as (1M
contract price + 150,000 contract modification + 200,000 bonus =
1,350,000).

PROBLEM 3: EXERCISES
1. Solutions:
Total contract price 20,000,000
(a) Costs incurred to date 2,000,000
Estimated costs to complete (squeeze) 14,000,000
(b) Estimated total contract costs 16,000,000
Expected profit (loss) 4,000,000
Multiply by: % of completion (a) ÷ (b) 12.50%
Profit (loss) to date 500,000
Profit recognized in prior years -
Profit (loss) for the year 500,000

Total contract price 20,000,000


Multiply by: % of completion 12.50%
Contract revenue to date 2,500,000
Contract revenue in prior years -
Contract revenue for the year 2,500,000
Cost of construction (squeeze) (2,000,000)
Profit (loss) for the year 500,000

2. Solutions:
Total contract price 4,500,000
(a) Costs incurred to date 1,350,000
Estimated costs to complete (given) 2,700,000
(b) Estimated total contract costs 4,050,000
Expected profit (loss) 450,000
Multiply by: % of completion (a) ÷ (b) 33 1/3%
Profit (loss) to date 150,000
Profit recognized in prior years -
Profit (loss) for the year 150,000

Total contract price 4,500,000


Multiply by: % of completion 33 1/3%
6
Contract revenue to date 1,500,000
Contract revenue in prior years -
Contract revenue for the year 1,500,000
Cost of construction (squeeze) (1,350,000)
Profit (loss) for the year 150,000

3. Solutions:
Requirement (a):
Total contract price 1,200,000
(a) Costs incurred to date 590,000
Estimated costs to complete (given) 410,000
(b) Estimated total contract costs 1,000,000
Expected profit (loss) 200,000
Multiply by: % of completion (a) ÷ (b) 59%
Profit (loss) to date 118,000
Profit recognized in prior years -
Profit (loss) for the year 118,000

Total contract price 1,200,000


Multiply by: % of completion 59%
Contract revenue to date 708,000
Contract revenue in prior years -
Contract revenue for the year 708,000
Cost of construction (squeeze) (590,000)
Profit (loss) for the year 118,000

Requirement (b):
Costs incurred 590,000
Profit recognized 118,000
Construction in progress 708,000

4. Solutions:
Requirement (a):

Contract revenue for the year (equal to cost incurred) 590,000


Cost of construction (squeeze) (590,000)
Profit (loss) for the year -

Requirement (b):
Costs incurred 590,000
Profit recognized -
Construction in progress 590,000

5. Solutions:
Requirement (a):
7
Contract revenue for the year -
Cost of construction -
Profit (loss) for the year -

Requirement (b):
Costs incurred 590,000
Profit recognized -
Construction in progress 590,000

6. Solutions:
20x1 20x2
Total contract price 6,000,000 6,000,000
(a) Costs incurred to date 2,250,000 4,800,000
Estimated costs to complete 2,250,000 -
(b) Estimated total contract costs 4,500,000 4,800,000
Expected profit (loss) 1,500,000 1,200,000
Multiply by: % of completion (a) ÷ (b) 50% 100%
Profit (loss) to date 750,000 1,200,000
Profit recognized in prior years - (750,000)
Profit (loss) for the year 750,000 450,000

20x1 20x2
Total contract price 6,000,000 6,000,000
Multiply by: % of completion 50% 100%
Contract revenue to date 3,000,000 6,000,000
Contract revenue in prior years - (3,000,000)
Contract revenue for the year 3,000,000 3,000,000
Cost of construction (squeeze) (2,250,000) (2,550,000)
Profit (loss) for the year 750,000 450,000

7. Solutions:
20x1 20x2
Contract revenue to date (a) 2,250,000 6,000,000
Contract revenue in prior years - (2,250,000)
Contract revenue for the year 2,250,000 3,750,000
Cost of construction (b) (2,250,000) (2,550,000)
Profit (loss) for the year - 1,200,000

(a)The contract revenue to date in 20x1 is equal to the ₱2,250,000 costs


incurred during that year. The contract revenue to date in 20x2 is equal to the
₱6,000,000 contract price because the construction is 100% complete.

(b)
Equal to the costs incurred during the year.

8
8. Solutions:
20x1 20x2
Contract revenue to date (a) - 6,000,000
Contract revenue in prior years - -
Contract revenue for the year - 6,000,000
Cost of construction (b) - (4,800,000)
Profit (loss) for the year - 1,200,000

(a) No revenue is recognized during the construction period because the

performance obligation is satisfied at a point in time. The whole of the


transaction price is recognized as revenue only in 20x2 when the construction
is completed.

(b) The costs incurred during the construction period are deferred and

recognized in full only in 20x2 when the related revenue is recognized.

9. Solutions:
20x1 20x2
Construction in progress, ending balances 122,000 364,000
Contract costs incurred to date (a) (105,000) (297,000)
Profit to date 17,000 67,000
Profit in previous years - (17,000)
Profit for the year 17,000 50,000

(a)
The contract costs incurred to date in 20x2 is computed as follows: (105,000 +
192,000 = 297,000).

20x1 20x2
Revenue for the year (squeeze) 122,000 242,000
Cost of construction (equal to costs incurred each yr.) (b) (105,000) (192,000)
Profit for the year 17,000 50,000

(b)
Under the ‘cost-to-cost’ method of measuring progress, the “cost of
construction” each year is equal to the contract cost incurred during the year.

Requirement (b):
Solution:
Progress billings, 20x2 420,000
Receivable, 20x2 (300,000)
Total collections 120,000

10. Solution:
The costs incurred to date are computed as follows:
20x1 20x2
(a) Costs incurred to date (squeeze) 978,750 4,524,000
Estimated costs to complete ignored ignored
(b) Estimated cost at completion (given) 6,525,000 6,960,000
9
(a) ÷ (b) Percentage of completion (given) 15% 65%

The costs of construction are computed as follows:


20x1 20x2
Costs incurred to date 978,750 4,524,000
Costs incurred in previous years - (978,750)
Costs incurred during the year 978,750 3,545,250

The profits are computed as follows:


20x1 20x2
Total contract price (given) 8,700,000 8,700,000
(a) Costs incurred to date (ignored)
Estimated costs to complete (ignored)
(b) Estimated cost at completion (given) 6,525,000 6,960,000
Expected profit (loss) 2,175,000 1,740,000
Multiply by: % of completion (given) 15% 65%
Profit (loss) to date 326,250 1,131,000
Profit recognized in prior years - (326,250)
Profit (loss) for the year 326,250 804,750

11. Solution:
Contract 1 Contract 2
Contract price 420,000 300,000
Costs incurred during the year 240,000 280,000
Estimated costs to complete 120,000 40,000
Total expected contract costs 360,000 320,000
Expected loss - (20,000)

Answer: Red Hot Co. recognizes a loss of ₱20,000 in 20x1. The loss is
recognized as a provision for onerous contract in accordance with PAS 37.

12. Solution:
Contract 1 Contract 2
Total contract price 420,000 300,000
(a) Costs incurred to date 240,000 280,000
Estimated costs to complete 120,000 40,000
(b) Estimated total contract costs 360,000 320,000
Expected profit (loss) 60,000 (20,000)
Multiply by: % of completion (a) ÷ (b) 66.67% N/A
Profit (loss) to date 40,000 (20,000)
Profit recognized in prior years - -
Profit (loss) for the year 40,000 (20,000)

Answer: Red Hot Co. recognizes a net profit of ₱20,000 (40,000 profit –
20,000 loss) in 20x1. The loss is recognized as a provision for onerous
contract in accordance with PAS 37.
10
13. Solution:
Contract 1 Contract 2
Contract price 420,000 300,000
Costs incurred during the year 240,000 280,000
Estimated costs to complete 120,000 40,000
Total expected contract costs 360,000 320,000
Expected loss - (20,000)

Answer: Red Hot Co. recognizes a loss of ₱20,000 in 20x1. The loss is
recognized as a provision for onerous contract in accordance with PAS 37.

14. Solutions:
Requirement (a):
Contract Contract Contract
Total
1 2 3
Total contract price 500,000 700,000 250,000
Costs incurred to date (a) 375,000 100,000 100,000
Estd. costs to complete - 400,000 100,000
Estd. total contract costs
375,000 500,000 200,000
(b)
Expected profit (loss) 125,000 200,000 50,000
% of completion (a) ÷ (b) 100% 20% 50%
Profit (loss) to date 125,000 40,000 25,000
Profit in prior years - - -
Profit (loss) for the yr. 125,000 40,000 25,000 190,000

Contract Contract Contract


Totals
1 2 3
Total contract price 500,000 700,000 250,000
Multiply by: % of completion 100% 20% 50%
Contract revenue to date 500,000 140,000 125,000
Revenue in prior years - - -
Revenue for the yr. 500,000 140,000 125,000 765,000
Costs incurred (squeeze) (375,000) (100,000) (100,000) (575,000)
Profit (loss) for the year 125,000 40,000 25,000 190,000

11
Requirement (b):
Contract 1 Contract 2 Contract 3 Totals
Costs incurred 375,000 100,000 100,000
Profit (loss) for the year 125,000 40,000 25,000
Total 500,000 140,000 125,000
Closing entry (a) (500,000) - -
Balance - 140,000 125,000 265,000

(a)
The CIP balance of Contract 1 is zeroed out because it is already
complete.

15. Solution:
Contract Contract Contract
Totals
1 2 3
Revenue for the yr. 500,000 - - 500,000
Costs incurred (squeeze) (375,000) - - (375,000)
Profit (loss) for the year 125,000 - - 125,000

16. Solution:
Contract Contract Contract
Totals
1 2 3
Revenue for the yr. 500,000 100,000 100,000 700,000
Costs incurred (squeeze) (375,000) (100,000) (100,000) (575,000)
Profit (loss) for the year 125,000 - - 125,000

The revenues recognized in contracts 2 and 3 are equal to the costs incurred
on those contracts during the year.

17. Solutions:

Requirement (a):
20x1 20x2
Contract revenue for the year (squeeze) 3,000,000 3,000,000
Cost of construction (2,250,000) (2,550,000) (a)
Profit (loss) for the year 750,000 450,000 (b)

(a)
(4.8M – 2.250M = 2.550M)
(b)
(1.2M – .750M = .450M)

Requirement (b):
Since the contract is 100% complete in 20x2, the transaction price is equal to
the sum of the revenues recognized in 20x1 and 20x2, i.e., 6,000,000 (3M +
3M).

12
18. Solution:
20x1 20x2
Total contract price 3,000,000 3,000,000
Multiply by: % of completion 20% 60%
Contract revenue to date 600,000 1,800,000
Contract revenue in prior years - (600,000)
Contract revenue for the year 600,000 1,200,000
Cost of construction (squeeze) (450,000) (990,000)
Profit (loss) for the year 150,000 210,000(a)

(a)
(360,000 -150,000) = 210,000

19. Solutions:
Requirement (a): Profit in 20x2

20x1 20x2
Total contract price 3,000,000 3,000,000
(a) Costs incurred to date 1,800,000
Estimated costs to complete, Dec. 31, 20x2 600,000
(b) Estimated total contract costs 2,250,000 2,400,000
Expected profit (loss) 750,000 600,000
Multiply by: % of completion (a) ÷ (b) 40% 75%
Profit (loss) to date 300,000 450,000
Profit recognized in prior years (given) - (300,000)
Profit (loss) for the year 300,000 150,000

Requirement (b): Estimated costs to complete – Dec. 31, 20x1

20x1
(a) Costs incurred to date (3rd step) – (2.250M x 40%) 900,000
Estimated costs to complete (Last step) – (squeeze) 1,350,000
(b) Estimated cost at completion (2nd step) – (given) 2,250,000
(a) ÷ (b) Percentage of completion (1st step) – (given) 40%

Requirement (c): Contract costs incurred in 20x2

Costs incurred to date, Dec. 31, 20x2 (given) 1,800,000


Costs incurred in 20x1 (see solution in 'b' above) (900,000)
Costs incurred in 20x2 900,000

Requirement (d): Revenues and Costs of construction – 20x1 & 20x2

20x1 20x2

13
Total contract price 3,000,000 3,000,000
Multiply by: % of completion (see ‘a’ above) 40% 75%
Contract revenue to date 1,200,000 2,250,000
Contract revenue in prior years - (1,200,000)
Contract revenue for the year 1,200,000 1,050,000
Cost of construction (see ‘b’ and ‘c’ above) (900,000) (900,000)
Profit (loss) for the year 300,000 150,000

20. Solutions:
Requirement (a):
Transaction price 20,000,000
Costs incurred to date, Dec. 31, 20x3 (squeeze) (18,400,000)
Profit to date, Dec. 31, 20x3 (400K + 1.4M - 200K) 1,600,000

Costs incurred to date, Dec. 31, 20x3 18,400,000


Costs incurred in 20x1 & 20x3 (3.6M + 8.2M) (11,800,000)
Costs incurred in 20x2 6,600,000

Requirement (b):
20x1 20x2
Total contract price (given) - Step 6 20,000,000
% of completion - (12M ÷ 20M) - Last step 60%
Contract revenue to date (squeeze) - Step 4 4,000,000 12,000,000
Contract revenue in prior years - Step 5 - (4,000,000)
Contract revenue for the year (squeeze) - Step 3 4,000,000 8,000,000
Costs incurred each year (see 'a' above) - Step 2 (3,600,000) (6,600,000)
Profit for the year (given) - Step 1 400,000 1,400,000

Requirement (c):
20x1
(a) Costs incurred to date (3.6M + 6.6M) (2nd step) 10,200,000
Estimated costs to complete (squeeze) - (Last step) 6,800,000
(b) Estimated cost at completion (10.2M ÷ 60%) (3rd step) 17,000,000
(a) ÷ (b) Percentage of completion (see ‘b’ above) - (1st step) 60%

21. Solution:
20x1 20x2 20x3
Total contract price 10,000,000 9,500,000 9,500,000
Costs incurred to date (a) 3,000,000 6,500,000 8,200,000
Estimated costs to complete 5,000,000 1,600,000 -
Estimated total contract costs (b) 8,000,000 8,100,000 8,200,000

14
Expected profit (loss) 2,000,000 1,400,000 1,300,000
% of completion (a) ÷ (b) 37.50% 80.25% 100.00%
Profit (loss) to date 750,000 1,123,500 1,300,000
Profit recognized in prior years - (750,000) (1,123,500)
Profit (loss) for the year 750,000 373,500 176,500

20x1 20x2 20x3


Total contract price 10,000,000 9,500,000 9,500,000
Multiply by: % of completion 37.50% 80.25% 100%
Contract revenue to date 3,750,000 7,623,750 9,500,000
Contract revenue in prior years - (3,750,000) (7,623,750)
Contract revenue for the year 3,750,000 3,873,750 1,876,250
Costs incurred each year (given) (3,000,000) (3,500,000) (1,700,000)
Profit (loss) for the year 750,000 373,750 176,250

The differences in the profits (20x2 and 20x3) are due to the
rounding-off of the percentage of completion in 20x2.

22. Solution:
Requirement (a):
20x1
Expected gross profit (given) 5,000,000
Multiply by: % of completion (given) 50.00%
Profit to date 2,500,000
Profit in previous years -
Profit for the year 2,500,000

Requirement (b):
Collection from mobilization fee (20M x 5%) 1,000,000

Unadjusted progress billings (20M x 50%) 10,000,000


Billing accepted in the following yr. (20M x 10%) (2,000,000)
Billing due in the following year (20M x 8%) (1,600,000)
Adjusted progress billings 6,400,000
Multiply by: (100% - 10% retention) 90%
Collections from progress billings 5,760,000
Total collections in 20x1 6,760,000

15
PROBLEM 4: CLASSROOM ACTIVITIES

ACTIVITY #1:

Solutions:

Step 1: Identify the contract with the customer

Requirement (a): YES, the contract qualifies for accounting under PFRS 15
because all of the requirements of “Step 1” are met.

a. The contract is approved and the parties are committed to perform their
respective obligations;
b. Each party’s rights regarding the goods or services to be transferred can
be identified from the contract;
c. The payment terms for the goods or services to be transferred can be
identified from the contract;
d. The contract has commercial substance; and
e. The consideration in the contract is probable of collection.

Step 2: Identify the performance obligations in the contract

Analysis: Performance obligations


There are three (3) promises in the contract: the promise to transfer the lot, to
provide the house design, and to construct the house.

However, these promises are not distinct on their own but rather a distinct
bundle of goods and services because of the following reasons:
a. The customer cannot benefit from the lot, the house design, and the
house separately because the contract requires the customer to
purchase those goods and services as a bundle. Moreover, Entity X
does not regularly sell those goods and services separately.
b. Each promise is not separately identifiable from the other promises in
the contract. This is because:
i. Each good or service is an input to a combined output specified by
the customer.
ii. Each good or service significantly modifies another good or service
promised in the contract.
iii. Each good or service is highly interrelated with the other goods or
services promised in the contract. For example, the customer’s
decision of not purchasing the house affects its ability to purchase
the lot.

Conclusion:
Requirement (b):
The promises to transfer the lot, the house design and the house shall be
combined and treated as a single performance obligation.

16
Analysis: Satisfaction of performance obligations

A performance obligation is satisfied over time if one of the following criteria


is met:
a. The customer simultaneously receives and consumes the benefits
provided by the entity’s performance as the entity performs.

b. The entity’s performance creates or enhances an asset (e.g., work in


progress) that the customer controls as the asset is created or
enhanced.

 Criteria (a) and (b) are not met because the following reasons:
a. Entity X retains control of the lot, the design, and the house
during the construction period. This precludes the customer
from simultaneously receiving and consuming the benefits
provided by the entity’s performance as the entity performs.
b. In case of default, Entity X forfeits the properties in its favor.

c. The entity’s performance does not create an asset with an alternative


use to the entity and the entity has an enforceable right to payment for
performance completed to date.

 Criterion (c) is not met because the following reasons:


a. The entity’s performance creates an asset with an alternative
use. This is evidenced by the fact that Entity X can resell
forfeited properties without much modification because the
design is standard and not unique to the customer.
b. Entity X’s right to payment is not directly correlated with
performance completed to date, i.e., the monthly payments are
due irrespective of the stage of completion of the house.

Conclusion:
Requirement (c):
The performance obligation is satisfied at a point in time.

Step 3: Determine the transaction price

Analysis:
The transaction price includes a variable consideration because of the
stipulated penalty (i.e., a reduction of 2% of contract price for every month of
delay in the completion of the construction). However, since Entity X does not
expect any delays on the construction, Entity X is not required to estimate the
variable consideration. This holds true until there is a subsequent change in
circumstances, in which case Entity X will be required to estimate the variable
consideration.

17
Conclusion:
Requirement (d):
The transaction price is ₱6,000,000. Using the practical expedient allowed
under PFRS 15, Entity X need not discount the installment payments
because they are due within 1 year.

Step 4: Allocate the transaction price to the performance obligations

Requirement (e):
Since the promises are treated as a distinct bundle of goods and services,
there is no need to allocate the transaction price to each of those promises.
Instead, the transaction price is allocated in its entirety to the single
performance obligation of transferring the lot together with the house
design and the house.

Step 5: Recognize revenue when (or as) the entity satisfies a performance
obligation

Requirement (d):
Since the performance obligation is satisfied at a point in time, revenue shall
be recognized at the point in time when control over the property (i.e., lot,
house design, and house) is transferred to the customer and the customer
accepts the property (i.e., the constructed house meets the specifications in
the contract).

Requirement (e):
Apr. Cash (6M x 20%) 1,200,000
1,
Receivable (6M x 80%) 4,800,000
20x1
Contract liability 6,000,000

“Receivable” is debited instead of “Contract asset” because Entity X has an


unconditional right to the consideration. This is because:
a. The contract is non-cancellable and the installment payments are
not dependent on performance completed to date.
b. The installment payments are conditioned only on the passage of
time (i.e., the installment payments are due at each month-end).
PFRS 15 states that, “A right to consideration is unconditional if only
the passage of time is required before payment of that consideration
is due, even if the amount is subject to refund in the future.”

Month-end entries:
Every Cash (6M x 80%) ÷ 18 months 266,666.67
end of 266,666.67
Receivable
the
month

18
Apr. Contract liability 6,000,000
1, 6,000,000
Revenue
20x2

ACTIVITY #2:

Solutions:

Step 1: Identify the contract with the customer

Requirement (a): YES, the contract qualifies for accounting under PFRS 15
because all of the requirements of “Step 1” are met.

a. The contract is approved and the parties are committed to perform their
respective obligations;
b. Each party’s rights regarding the services to be transferred can be
identified from the contract;
c. The payment terms for the services to be transferred can be identified
from the contract;
d. The contract has commercial substance; and
e. The consideration in the contract is probable of collection.

Step 2: Identify the performance obligations in the contract

Analysis: Performance obligations

The contract includes the promises to provide the construction services and
the designs (architectural, engineering, electrical, plumbing and other
necessary designs).

However, these promises are not distinct on their own but rather a distinct
bundle of services because of the following reasons:
a. Each promise is not separately identifiable from the other promises in
the contract. This is because:
i. Each service is an input to a combined output specified by the
customer.

Indicators:
 The designs constitute an integral part of the contract (see
ARTICLE 6 of the contract).
 The customer is precluded from subcontracting any of the
specific works that constitute the contract (see ARTICLE 9 of
the contract).
 The contract does not indicate separate billings for each of the
design works stated in the contract (see ‘bill of materials’).

19
ii. Each good or service significantly modifies another good or service
promised in the contract.

Indicator:
 A change in any of the design works would affect the
construction work.

iii. Each good or service is highly interrelated with the other goods or
services promised in the contract.

Indicators:
 See indicators in (a.i) above.
 Since the customer is precluded from subcontracting any of the
works specified in the contract, the customer’s decision of not
acquiring a specific work from the contractor affects the other
services covered in the contract. For example, if the customer
does not acquire the designs from Entity Y, Entity Y will not
perform the construction services, and vice-versa.

b. Although the customer can benefit from each of the promised services
(Entity Y regularly sells those services separately), the customer’s ability
to benefit from those services individually is limited because of the
reasons stated in (a) above.

Conclusion:
Requirement (b):
The promises to provide the designs and construction service shall be
combined and treated as a single performance obligation.

Analysis: Satisfaction of performance obligations

A performance obligation is satisfied over time if one of the following criteria


is met:
a. The customer simultaneously receives and consumes the benefits
provided by the entity’s performance as the entity performs.

b. The entity’s performance creates or enhances an asset (e.g., work in


progress) that the customer controls as the asset is created or
enhanced.

 Criteria (a) and (b) are met because, although Entity Y has the right
to supervise the construction activity, the customer retains
ownership over any structure built on the lot. This is evidenced by
the fact that, in case the contract is cancelled, any progress on the
contract inures to the benefit of the customer.

20
c. The entity’s performance does not create an asset with an alternative
use to the entity and the entity has an enforceable right to payment for
performance completed to date.

 Criterion (c) is met because of the following reasons:


a. In case the contract is cancelled, any structure built inures to
the benefit of the customer. Therefore, any asset created has
no alternative use to Entity Y.

Moreover, the lot belongs to the customer. Even if Entity Y


retains ownership over any structure built on the lot, Entity Y
would incur significant losses to direct the asset for another use
in case the contract is cancelled.

b. Entity Y has an enforceable right to payment for performance


completed to date. This is evidenced by the following:
i. Subsequent billings are based on Entity Y’s progress on
the contract.
ii. If the contract is cancelled, Entity Y has the right to
payment for any progress on the contract.

Conclusion:
Requirement (c):
The performance obligation is satisfied over time because the criteria above
are met.

Step 3: Determine the transaction price


Requirement (d):
The transaction price is equal to the fixed fee of ₱8,000,000.

Entity Y does not need to discount the transaction price because the timing of
agreed payments do not provide either the customer or Entity Y with a
significant benefit of financing, i.e., the payments on quarterly billings are due
within a short period of time.

Step 4: Allocate the transaction price to the performance obligations

Requirement (e):
Since the promises are treated as a distinct bundle of goods and services,
there is no need to allocate the transaction price to each of those promises.
Instead, the transaction price is allocated in its entirety to the single
performance obligation of completing the construction of the house in
accordance with the agreed specifications.

21
Step 5: Recognize revenue when (or as) the entity satisfies a performance
obligation

Requirement (f):
Since the performance obligation is satisfied over time, revenue shall be
recognized over the construction period based on Entity Y’s measure of its
progress towards the complete satisfaction of the performance obligation.

Entity Y shall determine an appropriate method for measuring its progress on


the contract. Because of insufficient information given in the problem, the
appropriate measure of progress is presumed to be the “cost-to-cost”
method, an application of the inputs method.

Requirement (g):
Sept. Cash (8M x 15%) 1,200,000
1, 1,200,000
Contract liability
20x1
to record the mobilization fee

Oct. 1 to Construction in progress 2,422,000


Dec. 31, 2,422,000
Cash (or other appropriate accounts)
20x1
to record the contract costs

The percentage of completion as of December 31, 20x1 is computed as


follows:

 The gross profit earned in 20x1 is computed as follows:

Total contract price 8,000,000


(a) Costs incurred to date 2,422,000
Estimated costs to complete (squeeze) 4,498,000
(b) Estimated total contract costs (see ‘bill of materials’) 6,920,000
Expected gross profit from contract 1,080,000
Multiply by: Percentage of completion (a) ÷ (b) 35%
Gross profit earned to date 378,000
Less: Gross profit earned in previous years -
Gross profit for the year 378,000

 The revenue and cost of construction in 20x1 are computed as


follows:

Total contract price 8,000,000


Multiply by: Percentage of completion 35%
Revenue to date 2,800,000
Less: Revenue recognized in previous yrs. -
Revenue for the year 2,800,000

22
Cost of construction (squeezed) (2,422,000)
Gross profit for the year (see computation above) 378,000

The year-end adjusting entry to recognize revenue is as follows:


Dec. Cost of construction 2,422,000
31, Construction in progress (gross profit) 378,000
20x1
Revenue 2,800,000

Dec. Contract liability (the mobilization fee) 1,200,000


31,
Receivable (squeeze) 1,600,000
20x1
Progress billings (8M x 35%) 2,800,000
to record the first quarterly progress billing

“Receivable” is debited instead of “Contract asset” because Entity Y has an


unconditional right to consideration for progress made on the contract (see
discussion in ‘Step 2’ above).

Dec. Cash [(2.8M – 1.2M) x 90%]* 1,440,000


31,
Receivable 1,440,000
20x1
to record the collection on the first quarterly
progress billing

* Total progress billing 2,800,000


Less: Mobilization fee (1,200,000)
Progress billing, net of mobilization fee 1,600,000
Less: 10% retention on subsequent billings (1.6M x 10%) (160,000)
Collection 1,440,000

Requirement (h):
Entity Y
Statement of financial position
As of December 31, 20x1

Current assets
Receivable (1,600,000 - 1,440,000) 160,000
Contract asset* -
Total current assets 160,000

Current liabilities
Contract liability* -
Total current liabilities -

23
*Construction in progress 2,800,000
Progress billing (2,800,000)
Contract asset/ Contract liability -

Entity Y
Statement of profit or loss
For the year ended December 31, 20x1

Revenue 2,800,000
Cost of construction (2,422,000)
Gross profit 378,000
Other operating expenses -
Profit for the year 378,000

PROBLEM 5: MULTIPLE CHOICE - THEORY


1. B 6. C 11. D
2. A 7. A 12. C
3. B 8. D 13. D
4. D 9. A 14. D
5. A 10. B 15. D

24
Chapter 8
Accounting for Franchise Operations –
Franchisor

PROBLEM 1: TRUE OR FALSE


1. FALSE 6. FALSE
2. FALSE 7. FALSE
3. FALSE 8. FALSE
4. FALSE 9. TRUE
5. FALSE 10. TRUE

PROBLEM 2: FOR CLASSROOM DISCUSSION


1 C 6 A
2 C 7 C
3 B 8 B
4 A 9 B
5 B 10 D
11. B
12. D

13. Solutions:

Step 2: Identify the performance obligations in the contract


There is only one performance obligation in the contract, i.e., the promise to
grant the license.

Since the promise to grant the license is distinct, the entity shall apply the
specific principles to determine whether the license provides the customer a
right to access or a right to use the entity’s intellectual property.

Analysis:
a. The contract requires ABC Co. to undertake activities that significantly
affect the intellectual property to which the customer has rights (i.e., ABC
Co. is continually involved in developing further the brand).
b. The customer is exposed to any positive or negative effects of those
activities.
c. Those activities do not result in the transfer of a good or a service to the
customer as those activities occur.

1
Conclusion:
The license provides the customer the right to access the entity’s intellectual
property as it exists throughout the license period. Therefore, the
performance obligation is satisfied over time.

Step 3: Determine the transaction price


The transaction price is the fixed payment of ₱1,400,000.

Step 4: Allocate the transaction price to the performance obligations


The ₱1,400,000 transaction price is allocated to the single performance
obligation of granting the license.

Step 5: Recognize revenue when (or as) a performance obligation is


satisfied

Since the performance obligation is satisfied over time, the entity recognizes
revenue over the license period by measuring its progress towards the
complete satisfaction of the performance obligation. The entity shall apply the
general principles of PFRS 15 to identify the method that best depicts its
performance in the license.

Because the contract provides the customer with unlimited use of the
licensed characters for a fixed term (i.e., 7 years), the most appropriate
measure of progress may be a time-based method (i.e., straight-line
method).

Journal entries:
Jan. 1, Cash on hand 1,400,000
20x1 Contract liability 1,400,000
to record the non-refundable initial
franchise fee
July 1,
20x1
No entry
Dec. 31, Contract liability (1.4M ÷ 7) x 6/12 100,000
20x1 Revenue 100,000
to recognize revenue from the franchise

PROBLEM 3: EXERCISES

1. Solutions:

Requirement (a):
Step 2: Identify the performance obligations in the contract
There is only one performance obligation in the contract, i.e., the promise to
grant the license.

The additional activities associated with the license (i.e., the creation of new
characters and the changes to the images of the characters) do not directly

2
transfer a good or service to the customer because they are part of the
entity’s promise to grant a license and, in effect, change the intellectual
property to which the customer has rights.

Since the promise to grant the license is distinct, the entity shall apply the
specific principles to determine whether the license provides the customer a
right to access or a right to use the entity’s intellectual property.

Analyses:
The problem states the following:
a. “However, newly created characters appear regularly and the images of
the characters evolve over time.”
b. “The contract requires the customer to use the latest images of the
characters.”

From the above statements, we can infer that the intellectual property to
which the customer has rights changes throughout the license period. This is
because new characters are continually created and that the images of the
characters are continually changed. Also, the contract requires the customer
to use the latest images of the characters.

Requirements (b) and (c):


Accordingly, the license provides the customer the right to access the
entity’s intellectual property as it exists throughout the license period.
Therefore, the performance obligation is satisfied over time.

Moreover, the following criteria under PFRS 15 are met:


a. The customer reasonably expects (arising from the entity’s customary
business practices) that the entity will undertake activities that will affect
the intellectual property to which the customer has rights (i.e., the
characters). Those activities include development of the characters and
the publishing of a weekly comic strip that includes the characters.
b. The rights granted by the license directly expose the customer to any
positive or negative effects of the entity’s activities because the contract
requires the customer to use the latest characters.
c. Even though the customer may benefit from those activities through the
rights granted by the license, they do not transfer a good or service to
the customer as those activities occur.

Requirement (d):
Step 5: Recognize revenue when (or as) a performance obligation is satisfied
Since the performance obligation is satisfied over time, the entity recognizes
revenue over the license period by measuring its progress towards the
complete satisfaction of the performance obligation. The entity shall apply the
general principles of PFRS 15 to identify the method that best depicts its
performance in the license.

Because the contract provides the customer with unlimited use of the
licensed characters for a fixed term (i.e., 4 years), the most appropriate
measure of progress may be a time-based method.

3
2. Solutions:
Requirement (a):
The only performance obligation in the contract is the promise to grant the
license.

Requirement (b):
The transaction price includes a variable consideration (i.e., sales-based
royalty).

Requirement (c):
The transaction price allocated to the single performance obligation of
granting the license.

Requirement (d):
Regardless of whether the license provides the customer the right to access
or the right to use the entity’s intellectual property, the entity recognizes
revenue as and when the ticket sales occur.

This is because the consideration for the license is a sales-based royalty and
the entity has already transferred the license to the movie to which the sales-
based royalty relates.

3. Solutions:
Requirement (a):
Step 2: Identify the performance obligations in the contract
There is only one performance obligation in the contract, i.e., the promise to
grant the license.

Since the promise to grant the license is distinct, the entity shall apply the
specific principles to determine whether the license provides the customer a
right to access or a right to use the entity’s intellectual property.

Analyses:
The problem states that “The customer can determine how and when to use
the right without further performance by Pongcuter Co. and does not expect
that Pongcuter Co. will undertake any activities that significantly affect the
intellectual property to which the customer has rights.”

From the statement above, it can be inferred that the intellectual property to
which the customer has rights will not change because the entity does not
undertake activities that significantly affect the intellectual property to which
the customer has rights.

Requirement (a.i):
Therefore, the nature of the entity’s promise in transferring the license is to
provide a right to use the entity’s intellectual property in the form and the

4
functionality with which it exists at the point in time that it is granted to the
customer.

Requirement (a.ii):
Consequently, the license is a performance obligation satisfied at a point in
time.

Requirement (b):
Step 3: Determine the transaction price
The transaction price is the fixed fee of ₱1,000,000.

Requirement (c):
Step 4: Allocate the transaction price to the performance obligations
The ₱1,000,000 transaction price is allocated to the single performance
obligation of granting the license.

Requirement (d):
Step 5: Recognize revenue when (or as) a performance obligation is satisfied
Pongcuter Co. recognizes the ₱1,000,000 fee as revenue on April 1, 20x1
when the customer has the ability to use the software.

Requirement (e):
Jan. 1, Cash on hand 1,000,000
20x1 Contract liability 1,000,000

Feb. 1,
20x1
No entry
Apr. 1, Contract liability 1,000,000
20x1 Revenue 1,000,000

4. Solutions:

Step 2: Identify the performance obligations in the contract


The promise to grant the license and the promise to transfer the equipment
are distinct because:
a. The customer can benefit from each promise on their own or together
with other resources that are readily available. (That is, the customer can
benefit from the license together with the equipment that is delivered
before the opening of the franchise and the equipment can be used in
the franchise or sold for an amount other than scrap value.)
b. The license and equipment are separately identifiable.

Moreover, the fact that ABC Co. regularly sells the license and the equipment
separately indicates that a customer can benefit from each of the license and
the equipment on its own or with other readily available resources.

Conclusion:
There are two separate performance obligations in the contract:

5
1. License; and
2. Equipment.

 Since the license is distinct, the entity applies the specific principles to
determine whether the license provides the customer the right to access
or the right to use the entity’s intellectual property.

The problems states that the license provides the customer the right to use
the entity’s intellectual property as it exists at the point in time at which the
license is granted. Therefore, the performance obligation of transferring the
license is satisfied at a point in time.

 ABC Co. uses the general principles to identify whether the performance
obligation of transferring the equipment is satisfied over time or at a point
in time.

Since control over the equipment transfers to the customer upon delivery, the
performance obligation is also satisfied at a point in time.

Summary of answers to Requirement (a):


The two separate performance obligations in the contract are as follows:
1. License (satisfied at a point in time)
2. Equipment (satisfied at a point in time)

Requirement (b):
Step 3: Determine the transaction price
The transaction price is sum of the 20% cash down payment and the present
value of the future cash flows from the note receivable. This is computed as
follows:

Cash down payment (100,000 x 20%) 20,000


PV of note receivable:
[(100K x 80%) ÷ 4] x PV of ordinary annuity @12%, n=4 60,747
Transaction price 80,747

Requirement (c):
Step 4: Allocate the transaction price to the performance obligations
The transaction price is allocated to the performance obligations in the
contract on the basis of their stand-alone selling prices. The allocation is
done as follows:

Performance Stand-alone Transaction


obligations selling prices Allocation price
(80,747 x
License 38,000 38K/78K) 39,338
(80,747 x
Equipment 40,000 40K/78K) 41,409
Totals 78,000 80,747

6
Requirement (d):
Step 5: Recognize revenue when (or as) a performance obligation is satisfied
The ₱41,409 allocated to the equipment will be recognized as revenue on
January 15, 20x1 while the ₱39,338 allocated to the license will be
recognized as revenue on February 1, 20x1.

Requirement (e):

The entry on January 1, 20x1 is as follows:


Jan. 1, Cash on hand 20,000
20x1 Note receivable 80,000
Contract liability 80,747
Unearned interest income 19,253

Jan. 15, Contract liability 41,409


20x1 Revenue 41,409
Jan. 15, Cost of sales 30,000
20x1 Inventory 30,000

The entry on March 1, 20x1 is as follows:


Feb. 1, Contract liability 39,338
20x1 Revenue 39,338

PROBLEM 4: CLASSROOM ACTIVITY


1. A
2. A
3. B
4. A
5. D (400,000 ÷ 5) = 80,000 x 1/12 = 6,666.67

PROBLEM 5: THEORY
1. D 6. B
2. C 7. A
3. D 8. B
4. B 9. B
5. D 10. D

7
Chapter 9
Consignment Sales

PROBLEM 1: TRUE OR FALSE


1. TRUE
2. TRUE
3. FALSE
4. FALSE
5. FALSE
6. TRUE
7. FALSE
8. FALSE
9. TRUE
10. FALSE

PROBLEM 2: FOR CLASSROOM DISCUSSION


1. A
2. D
3. B
4. A
5. Solutions:

Requirement (a):

The commission expense is computed as follows:

Net remittance 232,000


Freight out 16,000
Installation costs 8,000
Total 256,000
Divide by: 80%
Gross selling price of goods sold 320,000
Multiply by: 20%
Commission expense 64,000

Cost of goods sold is computed as follows:

Unit cost 10,000


Freight cost per unit (3,000 ÷ 20) 150
Total unit cost 10,150
Multiply by: Number of water heaters sold 16
Cost of goods sold 162,400

1
Profit is computed as follows:

Gross selling price of goods sold 320,000


Cost of goods sold (162,400)
Gross profit 157,600
Freight out (16,000)
Installation costs (8,000)
Commission expense (64,000)
Profit 69,600

Requirement (b):

Unit cost 10,000


Freight cost per unit (3,000 ÷ 20) 150
Total unit cost 10,150
Multiply by: Unsold units (20 - 16) 4
Ending inventory 40,600

PROBLEM 3: EXERCISE

Solutions:

Requirement (a):

The publisher’s suggested retail price is computed as follows:

Let X = Book sales at the publisher’s suggested retail price

2%X + 20%X = 69,300


20%X = 69,300
X = 69,300 / 22%
X = 315,000

315,000 ÷ 700 books sold = 450 publisher’s suggested retail price per book

The publisher’s profit is computed as follows:

Revenue (450 x 700) 315,000


Cost of goods sold (a) (225,400)
Gross profit 89,600
Tax expense (2% x 315,000) (6,300)
Commission expense (20% x 315,000) (63,000)
Profit 20,300

2
(a)
The cost of goods sold is computed as follows:
No. of books sold 700
Unit cost 300
Total 210,000
Freight (22 x 700) 15,400
Cost of goods sold 225,400

Requirement (b):

Commission based on publisher's suggested retail price


(315,000 x 20%) 63,000
Mark up on publisher's suggested retail price
(315,000 x 15%) 47,250
Commission income 110,250

Requirement (c):

No. of unsold books 300


Unit cost before freight 300
Total 90,000
Freight (22 x 300) 6,600
Ending inventory 96,600

PROBLEM 4: CLASSROOM ACTIVITY


Solution:

Requirement (a):

Total sales [2,100,000 x (8-3)] 10,500,000


Cost of goods sold (a) (5,125,000)
Gross profit 5,375,000
Commission (b) (1,750,000)
Finder's fee (5% x 1,750,000) (87,500)
Delivery, installation and testing (50,000 x 5) - 5,000 scrap (245,000)
Profit 3,292,500

(a) Cost of goods sold is computed as follows:

Unit cost 1,000,000


Freight per machine (200,000 ÷ 8) 25,000
Total unit cost 1,025,000
Multiply by: No. of machines sold 5
Cost of goods sold 5,125,000

3
(b)
The commission is computed as follows:
We will use the following formula for bonus after bonus:

B = P – [P ÷ (1 + Br)]

Commission = Gross sales – [Gross sales ÷ (1 + Commission rate)]


Commission = 10,500,000 – [10,500,000 ÷ (1 + 20%)]
Commission = 10,500,000 – 8,750,000
Commission = 1,750,000

Requirement (b):
Total sales [2,100,000 x (8-3)] 10,500,000
Commission (1,750,000)
Finder's fee (87,500)
Delivery, installation and testing (50,000 x 5) - 5,000 scrap (245,000)
Net remittance 8,417,500

Requirement (c):
Unit cost before freight 1,000,000
Freight per machine (200,000 ÷ 8) 25,000
Total unit cost 1,025,000
Multiply by: No. of unsold machines 3
Ending inventory 3,075,000

PROBLEM 5: MULTIPLE CHOICE - THEORY


1. B
2. B
3. C
4. D
5. B

4
PROBLEM 6: MULTIPLE CHOICE - COMPUTATIONAL
1. A (See solution in the second requirement)

2. B
Solution

The total unit cost is computed as follows:


Cost of consigned goods (1M x 8) 8,000,000
Freight 200,000
Total goods available for sale (in pesos) 8,200,000
Divide by: TGAS (in units) 8
Total unit cost 1,025,000

The number of unsold units is computed as follows:


Ending inventory 3,075,000
Divide by: Total unit cost 1,025,000
Unsold units 3

The number of units sold is computed as follows:


TGAS (in units) 8
Unsold units (3)
No. of units sold 5

Profit is computed as follows:

Total sales (2,100,000 x 5) 10,500,000


Cost of goods sold (a) (5,125,000)
Gross profit 5,375,000
Commission (b) (1,750,000)
Finder's fee (87,500)
Delivery, installation and testing (50,000 x 5) - 5,000 scrap (245,000)
Profit 3,292,500

(a)
Cost of goods sold is computed as follows:
Total unit cost 1,025,000
No. of units sold 5
Cost of goods sold 5,125,000

(b)
The commission is computed as follows:
We will use the following formula for bonus after bonus:

B = P – [P ÷ (1 + Br)]

Commission = Gross sales – [Gross sales ÷ (1 + Commission rate)]


Commission = 10,500,000 – [10,500,000 ÷ (1 + 20%)]

5
Commission = 10,500,000 – 8,750,000
Commission = 1,750,000

3. A (20 x 1,600) = 32,000

4. C (505 – 5) x ₱100 x 90% = 45,000

5. D (8,500 ÷ 85%) = 10,000

6. B (5,000 + 7,000 + 50,000) = 62,000

7. D (18,000 + 900) = 18,900

8. D (40,000 x 40%) + 27,000 = 43,000

9. C
Solution:
Sales revenue (7,700 x 5) 38,500
Cost of goods sold (6,000 x 5) + (720 x 5/12) (30,300)
Gross profit 8,200
Commission based on sales net of commission (a) (3,500)
Marketing expense based on commission (3,500 x 10%) (350)
Delivery and installation (30 x 5) (150)
Profit 4,200

(a) We will use a formula similar to the formula of bonus after bonus:
38,500
Commission based on sales after commission = 38,500 -
1+10%
Commission based on sales after commission = 3,500

10. A
Solution:
Sales 38,500
Commission based on sales net of commission (3,500)
Marketing expense based on commission (3,500 x 10%) (350)
Delivery and installation (30 x 5) (150)
Net remittance to consignor 34,500

6
Chapter 10
Installment Sales Method

PROBLEM 1: TRUE OR FALSE


1. TRUE
2. FALSE ₱0.80 = (₱1 x 8/10)
3. TRUE ₱4 = (₱5 x 8/10)
4. TRUE
5. TRUE
6. TRUE
7. TRUE
8. TRUE
9. TRUE
10. FALSE 40%
11. FALSE (5 / .20) = 25
12. FALSE (100 – 25) = 75
13. FALSE 540
(DGP, end. 10 / 100 receivable, end. = 10% GPR)
(100 receivable, end. + 500 collections) = 600 sale x 90% cost ratio =
540 cost of sale

PROBLEM 2: FOR CLASSROOM DISCUSSION


1. D

2. Solutions:

Requirement (a):

20x1 ₱500,000 x 30% = 150,000


20x2 ₱300,000 x 30% = 90,000
20x3 ₱200,000 x 30% = 60,000

Requirement (b):

20x1 1M - 500,000 = 500,000


20x2 1M – 500K – 300K = 200,000
20x3 1M – 500K – 300K – 200K = 0

Requirement (c):

20x1 500,000, ending A/R x 30% = 150,000


20x2 200,000, ending A/R x 30% = 60,000

1
20x3 0, ending A/R x 30% = 0

3. D

4. B

5. C (200,000 ÷ 25%) = 800,000 ending A/R


1M sale price – 800,000 ending A/R = 200,000 collections during the year

6. D 1M sale price – 750K cost of sale = 250K total gross profit – 200K
deferred = 50K realized

7. D 1M sale price – 750K cost of sale = 250K total gross profit – 220K
realized = 30K deferred

8. A 1M sale price – 750K cost of sale = 250K total gross profit – 180K
realized = 70K deferred ÷ 25% = 280,000

9. B 1M sale price – 750K cost of sale = 250K total gross profit – 160K
realized = 90K deferred ÷ 25% = 360,000 ending A/R;

1M sale price – 360,000 ending A/R = 640,000 collection

10. Solution:

Requirement (a):
The fair value of the repossessed inventory is computed as follows:
Estimated selling price 12,000
Reconditioning costs (2,000)
Normal profit margin (year of repossession) (12K x 30%) (3,600)
Fair value of repossessed property 6,400

The gain or loss on repossession is computed as follows:

Date Inventory (at fair value) 6,400


Deferred gross profit (25K x 20%) 5,000
Loss on repossession (squeeze) 1,600
Installment account receivable 13,000

Requirements (b) and (c):


The collections in 20x2 from the 20x1 and 20x2 sales are computed
as follows:

2
Installment Installment
receivable - 20x1 receivable - 20x2
Beg. 90,000 13,000 Write-off Beg. - - Write-off
Collection Collection
47,000 (squeeze) Sale 240,000 60,000 (squeeze)
30,000 End. 180,000 End.

The profit in 20x2 is computed as follows:

Realized gross profit from:


- 20x1 sale (47K x 20%) 9,400
- 20x2 sale (60K x 30%) 18,000
Total realized gross profit in 20x2 – Requirement (b) 27,400
Loss on repossession (1,600)
Profit in 20x2 – Requirement (c) 25,800

11. D

12. Solutions:

Scenario 1

The amount of over (under) allowance is determined as follows:


Trade-in value granted to customer 4,000
Fair value of merchandise traded-in (4,000)
Over (Under) allowance -

Requirement (a):
The journal entry to record the sale is as follows:

Date Inventory – traded-in (at fair value) 4,000


Installment account receivable (squeeze) 12,000
Installment sale 16,000

Requirement (b):

The gross profit rate is computed as follows:

Installment sale price 16,000


(Over) Under allowance -
Adjusted installment sale price 16,000
Cost of sale (10,000)

3
Gross profit 6,000

Gross profit rate 37.5%

The realized gross profit is computed as follows:

Trade-in value granted to customer 4,000


(Over) under allowance -
Subsequent collections 6,000
Total collections on installment sale 10,000
Multiply by: Gross profit rate 37.5%
Realized gross profit - 20x1 3,750

Scenario 2

The amount of over (under) allowance is determined as follows:


Trade-in value granted to customer 4,000
Fair value of merchandise traded-in (3,000)
Over allowance 1,000

Requirement (a):
The journal entry to record the sale is as follows:
Date Inventory – traded-in (at fair value) 3,000
Over allowance on trade-in 1,000
Installment account receivable (squeeze) 12,000
Installment sale 16,000

Requirement (b):
The gross profit rate is computed as follows:
Installment sale price 16,000
Over allowance (1,000)
Adjusted installment sale price 15,000
Cost of sale (10,000)
Gross profit 5,000

Gross profit rate 33.33%

4
The realized gross profit is computed as follows:
Trade-in value granted to customer 4,000
(Over) under allowance (1,000)
Subsequent collections 6,000
Total collections on installment sale 9,000
Multiply by: Gross profit rate 33.33%
Realized gross profit - 20x1 3,000

Scenario 3

The amount of over (under) allowance is determined as follows:


Trade-in value granted to customer 4,000
Fair value of merchandise traded-in (6,000)
Under allowance (2,000)

Requirement (a):
The journal entry to record the sale is as follows:
Date Inventory – traded-in (at fair value) 6,000
Installment account receivable (squeeze) 12,000
Installment sale 16,000
Under allowance on trade-in 2,000

Requirement (b):
The gross profit rate is computed as follows:

Installment sale price 16,000


Under allowance 2,000
Adjusted installment sale price 18,000
Cost of sale (10,000)
Gross profit 8,000
Gross profit rate 55.56%

The realized gross profit is computed as follows:


Trade-in value granted to customer 4,000
(Over) under allowance 2,000
Subsequent collections 6,000
Total collections on installment sale 12,000
Multiply by: Gross profit rate 55.56%
Realized gross profit - 20x1 6,667

5
13. C

14. Solution:
Total collections from 20x1 sales 10,000
Cost of 20x1 sales (8,000)
Gross profit - 20x1 sales 2,000
Total collections from 20x2 sales 12,000
Cost of 20x2 sales (9,000)
Gross profit - 20x2 sales 3,000
Gross profit recognized in 20x2 5,000

15. A

PROBLEM 3: EXERCISES
1. Solutions:
Requirement (a):
Installment sales 1,000,000
Cost of sales (800,000)
Deferred gross profit - unadjusted balance 200,000

Requirement (b):
Installment sales 1,000,000
Installment accounts receivable - Dec. 31, 20x1 (600,000)
Collections in 20x1 400,000

Requirement (c):
Installment sales 1,000,000
Cost of sales (800,000)
Deferred gross profit - unadjusted balance 200,000
Gross profit rate based on sales (200K / 1M) 20%

Requirement (d):
Collections in 20x1 400,000
Multiply by: Gross profit rate based on sales 20%
Realized gross profit - 20x1 80,000

Requirement (e):
Deferred gross profit - unadjusted balance 200,000
Realized gross profit - 20x1 (80,000)
Deferred gross profit - adjusted balance 120,000

OR

6
Installment accounts receivable - Dec. 31, 20x1 600,000
Multiply by: Gross profit rate based on sales 20%
Deferred gross profit - adjusted balance 120,000

2. Solutions:
Requirement (a):
Deferred gross profit (before year-end adjustment) 200,000
Divide by: Gross profit based on sales 20%
Installment sales 1,000,000

Requirement (b):
Installment sales 1,000,000
Collections in 20x1 (400,000)
Installment accounts receivable - Dec. 31, 20x1 600,000

Requirement (c):
Collections in 20x1 400,000
Multiply by: Gross profit rate based on sales 20%
Realized gross profit - 20x1 80,000

Requirement (d):
Deferred gross profit (before year-end adjustment) 200,000
Realized gross profit - 20x1 (80,000)
Deferred gross profit - Dec. 31, 20x1 120,000

Reconciliations:
Collections in 20x1 400,000
Installment accounts receivable - Dec. 31, 20x1 600,000
Installment sales 1,000,000

Installment accounts receivable - Dec. 31, 20x1 600,000


Multiply by: Gross profit rate based on sales 20%
Deferred gross profit - Dec. 31, 20x1 120,000

Deferred gross profit - Dec. 31, 20x1 120,000


Divide by: Gross profit based on sales 20%
Installment accounts receivable - Dec. 31, 20x1 600,000

3. Solution:
Installment sales 900,000
Installment accounts receivable, Dec. 31, 20x1 500,000
Collections in 20x1 400,000
7
Multiply by: (100% - 60%) 40%
Realized gross profit - 20x1 160,000

4. Solutions:
Requirement (a):
Deferred gross profit, before year-end adjustment 560,000
Divide by: Gross profit on sales 40%
Total sales 1,400,000
Installment accounts receivable, Dec. 31, 20x1 (800,000)
Collections - 20x1 600,000

Requirement (b):
Collections - 20x1 600,000
Multiply by: Gross profit on sales 40%
Realized gross profit - 20x1 240,000

5. Solution:

20x1 installment accounts 16,250


Multiply by: Gross profit rate based on sales 30%/130%
Deferred gross profit (after adjustment) - 20x1 sales 3,750

20x2 installment accounts 90,000


Multiply by: Gross profit rate based on sales 33 1/3%/133 1/3%
Deferred gross profit (after adjustment) - 20x2 sales 22,500

Total deferred gross profit (after adjustment) 26,250

Deferred gross profit (before adjustment) 38,000


Deferred gross profit (after adjustment) (26,250)
Realized gross profit - 20x2 11,750
Expenses relating to installment sales (1,500)
Profit from installment sales - 20x2 10,250

6. Solutions:
Requirement (a):
The gross profit rates are computed as follows:
20x1 20x2 20x3
Installment sales 300,000 375,000 360,000
Cost of sales 225,000 285,000 252,000
Gross profit 75,000 90,000 108,000
Gross profit rate based on sales 25% 24% 30%

8
20x1 Deferred gross profit, Dec. 31, 20x3 -
Divide by: Gross profit rate 25%
20x1 Installment accounts receivable, Dec. 31, 20x3 -

20x2 Deferred gross profit, Dec. 31, 20x3 9,000


Divide by: Gross profit rate 24%
20x2 Installment accounts receivable, Dec. 31, 20x3 37,500

20x3 Deferred gross profit, Dec. 31, 20x3 72,000


Divide by: Gross profit rate 30%
20x3 Installment accounts receivable, Dec. 31, 20x3 240,000

Total installment accounts receivable, Dec. 31, 20x3 277,500

Requirement (b):
20x1 Deferred gross profit, Dec. 31, 20x2 15,000
Divide by: Gross profit rate 25%
20x1 Installment accounts receivable, Dec. 31, 20x2 60,000
20x1 Installment accounts receivable, Dec. 31, 20x3 -
Collection during 20x3 from 20x1 sales 60,000

20x2 Deferred gross profit, Dec. 31, 20x2 54,000


Divide by: Gross profit rate 24%
20x2 Installment accounts receivable, Dec. 31, 20x2 225,000
20x2 Installment accounts receivable, Dec. 31, 20x3 37,500
Collection during 20x3 from 20x2 sales 187,500

Installment sales - 20x3 360,000


20x3 Installment accounts receivable, Dec. 31, 20x3 240,000
Collection during 20x3 from 20x3 sales 120,000

Total collections during 20x3 367,500

Requirement (c):
Collection during 20x3 from 20x1 sales 60,000
Multiply by: Gross profit rate - 20x1 sales 25%
Realized gross profit in 20x3 from 20x1 sales 15,000

Collection during 20x3 from 20x2 sales 187,500


Multiply by: Gross profit rate - 20x2 sales 24%

9
Realized gross profit in 20x3 from 20x2 sales 45,000

Collection during 20x3 from 20x3 sales 120,000


Multiply by: Gross profit rate - 20x3 sales 30%
Realized gross profit in 20x3 from 20x3 sales 36,000

Total realized gross profit in 20x3 96,000

7. Solutions:
Requirement (a):
20x1 installment account receivable, Dec. 31, 20x2 112,500*
20x1 installment account receivable, Dec. 31, 20x3 (60,000)
Collections in 20x3 from 20x1 sales 52,500
Multiply by: Gross profit rate - 20x1 30%
Realized gross profit in 20x3 from 20x1 sales 15,750

20x2 installment account receivable, Dec. 31, 20x2 300,000


20x2 installment account receivable, Dec. 31, 20x3 (195,000)
Collections in 20x3 from 20x2 sales 105,000
Multiply by: Gross profit rate - 20x2 40%
Realized gross profit in 20x3 from 20x2 sales 42,000

20x3 installment sales 495,000


20x3 installment account receivable, Dec. 31, 20x3 (390,000)
Collections in 20x3 from 20x3 sales 105,000
Multiply by: Gross profit rate - 20x3 35%
Realized gross profit in 20x3 from 20x1 sales 36,750

Total realized gross profit - 20x3 94,500

* (135,000 less 22,500 unpaid balance in repossessed merchandise) =


112,500

Requirement (b):
20x3 Inventory (at fair value) 15,000
Deferred gross profit (22.5K x 30%) 6,750
Loss on repossession (squeeze) 750
Installment account receivable 22,500

10
8. Solution:
Cash down payment 600,000
Collection from installment payment (900K + 540K) 1,440,000
Total collections 2,040,000
Cost of sale (4,000,000)
Excess of collection over cost -

Since total collections do not exceed the cost of sale, no income shall be
recognized by Sound Co.

PROBLEM 4: CLASSROOM ACTIVITY


1. Solutions:

Requirement (a):

20x1 ₱400,000 x 25% = 100,000


20x2 ₱150,000 x 25% = 37,500
20x3 ₱ 50,000 x 25% = 12,500

Requirement (b):

20x1 600K – 400K = 200,000


20x2 600K – 400K – 150K = 50,000
20x3 600K – 400K – 150K – 50K = 0

Requirement (c):

20x1 200,000, ending A/R x 25% = 50,000


20x2 500,000, ending A/R x 25% = 12,500
20x3 0, ending A/R x 25% = 0

2. Solutions:
Case 1: 112,000 ÷ 35% = 320,000

Case 2: 269,500 ÷ 35% = 770,000 A/R, end


900,000 – 770,000 = 130,000

Case 3: 900K – 585K = 315K total gross profit – 200K deferred = 115,000
realized

Case 4: 300,000 x 35% = 105,000 realized


900K – 585K = 315K total gross profit – 105K realized = 210,000
deferred

11
Case 5: 900K – 585K = 315K total gross profit – 220K realized = 95,000
deferred

Case 6: 900K – 585K = 315K total gross profit – 180K realized = 135,000
deferred ÷ 35% = 385,714 A/R, end.

Case 7:
If the realized gross profit is ₱147,000, how much is the total collections
during the year?

147K ÷ 35% = 420,000 total collections

3. Solution:

Requirement (a):
The fair value of the repossessed inventory is computed as follows:
Estimated selling price 24,000
Reconditioning costs (4,000)
Normal profit margin (year of repossession) (24K x 30%) (7,200)
Fair value of repossessed property 12,800

The gain or loss on repossession is computed as follows:

Date Inventory (at fair value) 12,800


Deferred gross profit (25K x 20%) 10,000
Loss on repossession (squeeze) 3,200
Installment account receivable 26,000

Requirements (b) and (c):


The collections in 20x2 from the 20x1 and 20x2 sales are computed
as follows:

Installment Installment
receivable - 20x1 receivable - 20x2
Beg. 180,000 26,000 Write-off Beg. - - Write-off
Collection Collection
94,000 (squeeze) Sale 480,000 120,000 (squeeze)
60,000 End. 360,000 End.

The profit in 20x2 is computed as follows:

Realized gross profit from:

12
- 20x1 sale (94K x 20%) 18,800
- 20x2 sale (120K x 30%) 36,000
Total realized gross profit in 20x2 – Requirement (b) 54,800
Loss on repossession (3,200)
Profit in 20x2 – Requirement (c) 51,600

4. Solution:

Scenario 1

The amount of over (under) allowance is determined as follows:


Trade-in value granted to customer 8,000
Fair value of merchandise traded-in (8,000)
Over (Under) allowance -

Requirement (a):
The journal entry to record the sale is as follows:

Date Inventory – traded-in (at fair value) 8,000


Installment account receivable (squeeze) 24,000
Installment sale 32,000

Requirement (b):

The gross profit rate is computed as follows:

Installment sale price 32,000


(Over) Under allowance -
Adjusted installment sale price 32,000
Cost of sale (20,000)
Gross profit 12,000

Gross profit rate 37.5%

The realized gross profit is computed as follows:

13
Trade-in value granted to customer 8,000
(Over) under allowance -
Subsequent collections 12,000
Total collections on installment sale 20,000
Multiply by: Gross profit rate 37.5%
Realized gross profit - 20x1 7,500

Scenario 2
Solution:
The amount of over (under) allowance is determined as follows:
Trade-in value granted to customer 8,000
Fair value of merchandise traded-in (6,000)
Over allowance 2,000

Requirement (a):
The journal entry to record the sale is as follows:
Date Inventory – traded-in (at fair value) 6,000
Over allowance on trade-in 2,000
Installment account receivable (squeeze) 24,000
Installment sale 32,000

Requirement (b):
The gross profit rate is computed as follows:
Installment sale price 32,000
Over allowance (2,000)
Adjusted installment sale price 30,000
Cost of sale (20,000)
Gross profit 10,000

Gross profit rate 33.33%

The realized gross profit is computed as follows:


Trade-in value granted to customer 8,000
(Over) under allowance (2,000)
Subsequent collections 12,000
Total collections on installment sale 18,000
Multiply by: Gross profit rate 33.33%
Realized gross profit - 20x1 6,000

14
Scenario 3
Solution:

The amount of over (under) allowance is determined as follows:


Trade-in value granted to customer 8,000
Fair value of merchandise traded-in (12,000)
Under allowance (4,000)

Requirement (a):
The journal entry to record the sale is as follows:
Date Inventory – traded-in (at fair value) 12,000
Installment account receivable (squeeze) 24,000
Installment sale 32,000
Under allowance on trade-in 4,000

Requirement (b):
The gross profit rate is computed as follows:

Installment sale price 32,000


Under allowance 4,000
Adjusted installment sale price 36,000
Cost of sale (20,000)
Gross profit 16,000
Gross profit rate 55.56%

The realized gross profit is computed as follows:


Trade-in value granted to customer 8,000
(Over) under allowance 4,000
Subsequent collections 12,000
Total collections on installment sale 24,000
Multiply by: Gross profit rate 55.56%
Realized gross profit - 20x1 13,334

5. Solution:

Total collections from 20x1 sales 20,000


Cost of 20x1 sales (19,000)
Gross profit - 20x1 sales 4,000
Total collections from 20x2 sales 24,000

15
Cost of 20x2 sales (18,000)
Gross profit - 20x2 sales 6,000
Gross profit recognized in 20x2 10,000

PROBLEM 10-5: THEORY


1. C 6. C 11. C
2. C 7. B 12. A
3. C 8. A 13. B
4. A 9. B 14. C
5. B 10. B 15. D

PROBLEM 10-7: MULTIPLE CHOICE (COMPUTATIONAL)

Solutions:

1. A 200K – 60K = 140K realized ÷ 25% = 560,000

2. C

Date Collection Interest Amortization Principal


9/30/x1 48,000
9/30/x1 4,800 - 4,800 43,200
10/31/x1 4,800 432 4,368 38,832
11/30/x1 4,800 388 4,412 34,420
12/31/x1 4,800 344 4,456 29,965
Totals 19,200 1,165 18,035

18,035 x 37.5% = 6,763

3. A
Inventory 16,800
Deferred gross profit (29,965 x 37.5%) 11,237
Loss on repossession 1,928
Receivable 29,965

16
4. C

The 20x1 sale is computed as follows:

Date Collection Interest Amortization Principal


Date of sale 158,000 (squeeze)
20x1 71,000 7,000 64,000 94,000(a)
(squeeze) (given) (given) (start)
(a)
108,000 N/R – 14,000 discount on N/R = 94,000 carrying amount 12/31/x1

The cost of goods sold is computed as follows:

Inventory

beg. -
COGS
Purchases 100,850 90,850 (squeeze)

10,000 end.

The gross profit rate in 20x1 is computed as follows:

Sales 158,000
COGS (90,850)
Gross profit 67,150
Gross profit rate 42.50%

The realized gross profit rate is computed as follows:


Collections of principal 64,000
Gross profit rate 42.50%
Realized gross profit 27,200

5. B

The collection on principal of 20x1 receivable in 20x2 is computed as follows:

Date Collection Interest Amortization Principal


Date of sale 158,000
20x1 71,000 7,000 64,000 94,000

17
20x2 34,000 60,000 (b)
(squeeze) (start)
(b) 72,000 N/R – 12,000 discount on N/R = 60,000 carrying amount 12/31/x2

The collection on principal of 20x2 receivable in 20x2 is computed as follows:

Total collection on principal in 20x2 100,000


Less: Collection from 20x1 receivable (34,000)
Collection from 20x2 receivable 66,000

The 20x2 sale is computed as follows:

Date Collection Interest Amortization Principal


Date of sale 170,000 (squeeze)
(c)
20x2 66,000 104,000
(see above) (start)
(c)120,000 N/R – 16,000 discount on N/R = 104,000 carrying amount
12/31/x1

The cost of goods sold is computed as follows:

Inventory
beg. -
COGS
Purchases 105,250 89,250 (squeeze)
16,000 end.

The gross profit rate in 20x1 is computed as follows:

Sales 170,000
COGS (89,250)
Gross profit 80,750
Gross profit rate 47.50%

The realized gross profit rate is computed as follows:

From 20x2 sale:

Collections of principal from 20x2 sale 66,000


Gross profit rate – 20x2 47.50%
Realized gross profit in 20x2 from 20x2 sale 31,350

From 20x1 sale:


18
Collections of principal from 20x1 sale
34,000
(see amortization table above)
Gross profit rate – 20x1
42.50%
(see solution in previous problem)
Realized gross profit in 20x2 from 20x1 sale 14,450

Total realized gross profit in 20x2: (31,350 + 14,450) = 45,800

19
Chapter 11
Home office, Branch and Agency Accounting

PROBLEM 1: TRUE OR FALSE


1. FALSE
2. TRUE
3. TRUE
4. TRUE
5. FALSE
6. FALSE
7. FALSE
8. FALSE
9. FALSE
10. TRUE

PROBLEM 2: FOR CLASSROOM DISCUSSION

1. Solutions:
Requirement (a):
Home office books Branch books
Jan . 1, 20x1 Jan . 1, 20x1
Investment in branch……...500K Cash……………………...500K
Cash………………………….…500K Home office..…………… 500K

(a) (a)
Investment in branch……...100K Inventory…………………200K
Accounts payable……………100K Accounts payable 100K
Home office………………100K

(b) (b)
No entry Equipment………………120K
Cash……………………….120K

(c) (c)
Investment in branch……... 60K Equipment……………….60K
Accum. depreciation…..……300K Home office………………..60K
Equipment………………….. 360K

(d) (d)
No entry Cash ……………………..600K
Sales………………………600K

Cost of goods sold……...180K

1
(200K – 20K unsold)
Inventory…………………180K

(e) (e)
Cash…………………………..80K Home office……………80K
Investment in branch………..80K Cash……………………….80K

(f) (f)
Investment in branch………25K Expenses(150K + 25K) 175K
Expenses…………………….25K Depreciation expense…. 10K
Cash………………………150K
Accum. depn………………10K
Home office……………….25K

(g)Closing entries: (g) Closing entries:


Sales……………………..600K
Cost of goods sold………180K
Expenses…………………175K
Depreciation expense…….10K
Income summary……….. 235K

Investment in branch…..235K Income summary………235K


Income summary…………….235K Home office……………235K

Requirement (b):
Investment in branch Home office
Jan. 1 500,000 500,000 Jan. 1
(a) 100,000 100,000 (a)
(c) 60,000 80,000 (e) 80,000 60,000 (c)
(f) 25,000 25,000 (f)
(g) 235,000 235,000 (g)
840,000 840,000

Requirement (c):
Cash 750,000
Inventory 20,000
Equipment 180,000
Accum. Depreciation (10,000)
Total assets 940,000

Accounts payable 100,000


Home office 840,000
Total liabilities & equity 940,000

2
Sales 600,000
Cost of goods sold (180,000)
Gross profit 420,000
Expenses (175,000)
Depreciation expense (10,000)
Profit 235,000

2. Solution:
Home office Branch Combined
Cash 500,000 200,000 700,000
Accounts receivable 1,000,000 400,000 1,400,000
Inventory 680,000 300,000 980,000
Investment in branch 400,000 - -
Land 2,000,000 2,000,000
Building-net 4,000,000 4,000,000
Total assets 8,580,000 900,000 9,080,000

Accounts payable 4,000,000 500,000 4,500,000


Ordinary share capital 2,000,000 - 2,000,000
Share premium 200,000 - 200,000
Retained earnings 2,380,000 - 2,380,000
Home office 400,000 -
Total liabilities & equity 8,580,000 900,000 9,080,000

3. Solutions:

Requirement (a):
Home office books Branch books
(a) (a)
Investment in branch……...470K Shipments from HO…..450K
(300K x 150%) + 20K Freight-in……………… 20K
Shipments to branch…….. 300K Home office…………… 470K
Allowance for mark-up…… 150K
Cash………………………… 20K

(b) (b)
No entry Purchases……………..100K
Freight-in…………………2K
Cash……………………….102K

3
(c) (c)
No entry Cash…………………..500K
Sales………………………500K
(d) (d)
Inventory – end.,,,,,,, 235K
(470K x ½)
Income summary……..235K

Requirement (b):
Sales 500,000
Cost of goods sold:
Shipments from HO 450,000
Freight-in 22,000
Purchases 100,000
Ending inventory (235,000) (337,000)
Individual gross profit 163,000

Requirement (c):
Sales 500,000
Cost of goods sold:
Shipments from HO 300,000
Freight-in 22,000
Purchases 100,000
Ending inventory (160,000) (262,000)
Individual gross profit 238,000

Requirement (d):
150,000 allow. for markup x 50% sold = 75,000

4
PROBLEM 3: EXERCISES

1. Solutions:
Requirement (a):
Home office books Branch books
Jan . 1, 20x1 Jan . 1, 20x1
Investment in branch……...600K Cash……………………...600K
Cash………………………….…600K Home office..…………… 600K

(a) (a)
Investment in branch……...25K Prepaid supplies………100K
Accounts payable…………… 25K Accounts payable …………75K
Home office……………… 25K

(b) (b)
No entry Equipment………………80K
Cash……………………….80K

(c) (c)
Investment in branch……...120K Equipment……………….120K
Accum. depreciation…..…… 80K Home office………………120K
Equipment………………….. 200K

(d) (d)
Accounts payable……..25K No entry
Cash………………………….25K
(e) (e)
No entry Accounts payable ………50K
Cash……………………….50K

(f) (f)
No entry Cash ……………………..800K
Service fees………………800K

(g) (g)
Cash…………………………..180K Home office……………180K
Investment in branch………180K Cash……………………….180K

(h) (h)
Investment in branch………60K Expenses……………… 250K
Expenses…………………….60K Depreciation expense…. 40K
Advertising expense …….60K
Supplies expense………..95K
Cash………………………250K
Accum. depn………………40K
Home office……………….60K
Prepaid supplies…………. 95K

5
(i)Closing entries: (i) Closing entries:
Service fees…………….800K
Expenses……………… 250K
Depreciation expense…. 40K
Advertising expense …….60K
Supplies expense………..95K
Income summary……….. 355K
Investment in branch…..355K
Income summary…………….355K Income summary………355K
Home office……………355K

Requirement (b):
Investment in branch Home office
Jan. 1 600,000 600,000 Jan. 1
(a) 25,000 25,000 (a)
(c) 120,000 180,000 (g) 180,000 120,000 (c)
(h) 60,000 60,000 (h)
(i) 355,000 355,000 (i)
980,000 980,000

Requirement (c):
Cash 840,000
Prepaid supplies 5,000
Equipment 200,000
Accum. Depreciation (40,000)
Total assets 1,005,000

Accounts payable 25,000


Home office 980,000
Total liabilities & equity 1,005,000

Service fees 800,000


Expenses (250,000)
Depreciation expense (40,000)
Advertising expense (60,000)
Supplies expense (95,000)
Profit 355,000

6
2. Solution:
Home office Branch Combined
Cash 600,000 240,000 840,000
Accounts receivable 1,200,000 480,000 1,680,000
Inventory 816,000 360,000 1,176,000
Investment in branch 480,000 - -
Land 2,400,000 - 2,400,000
Building-net 4,800,000 - 4,800,000
Total assets 10,296,000 1,080,000 10,896,000
- - -
Accounts payable 4,800,000 600,000 5,400,000
Ordinary share capital 2,400,000 - 2,400,000
Share premium 240,000 - 240,000
Retained earnings 2,856,000 - 2,856,000
Home office - 480,000 -
Total liabilities & equity 10,296,000 1,080,000 10,896,000

3. Solutions:

Requirement (a):
Home office books Branch books
(a) (a)
Investment in branch……...500K Shipments from HO…..480K
(400K x 120%) + 20K Freight-in……………… 20K
Shipments to branch…….. 400K Home office…………… 500K
Allowance for mark-up…… 80K
Cash………………………… 20K

(b) (b)
No entry Purchases……………..80K
Freight-in…………………2K
Cash……………………….82K

(c) (c)
No entry Cash…………………..600K
Sales………………………600K
(d) (d)
Inventory – end…… 125K
(500K x ¼ )
Income summary……..125K

7
Requirement (b):
Sales 600,000
Cost of goods sold:
Shipments from HO 480,000
Freight-in 22,000
Purchases 80,000
Ending inventory (125,000) (457,000)
Individual gross profit 143,000

Requirement (c):
Sales 600,000
Cost of goods sold:
Shipments from HO 400,000
Freight-in 22,000
Purchases 80,000
Ending inventory (105,000) (397,000)
Individual gross profit 203,000

Requirement (d):
80,000 allow. for markup x ¾ sold = 60,000

4. Answer: 250,000 - Only the sales by the branch to outside parties. Intra-
company billings are eliminated in the combined financial statements.

8
PROBLEM 4: CLASSROOM ACTIVITIES

ACTIVITY #1:
Solutions:

Requirement (a):
Home office books Branch books
Jan . 1, 20x1 Jan . 1, 20x1
Investment in branch……...10M Cash……………………...10M
Cash………………………….… 10M Home office..…………… 10M

(a) (a)
Investment in branch……...30M Land………………………10M
Cash………………..………… 30M Building…………………..20M
Home office……………… 30M

(b) (b)
Investment in branch……20.5M Shipments from HO……20M
Shipments to the branch……..20M Freight-in………………..500K
Cash………………………… 500K Home office…………… 20.5M

(c) (c)
Investment in branch……... 5M Shipments from HO…… 5M
Shipments to the branch……. 5M Freight-in………………..100K
Home office……………… 5M
Cash………………………100K

(d) (d)
Equipment…………… 900K Home office……………900K
Investment in branch……900K Cash………………………900K

(e) (e)
No entry Furniture………… ……600K
Cash………………………600K

(f) (f)
No entry Purchases…………….. 10M
Accounts payable………..10M

(g) (g)
No entry Cash……………………50M
Accounts receivable….50M
Sales…………………….100M

(h) (h)
Cash…………………………10M Cash……………………30M
Investment in branch………10M Home office…………...10M
Accounts receivable……..40M

9
(i) (i)
Cash…………………….35M Home office……….35M
Home office………………….35M Cash…………………….35M

(j) (j)
No entry Accounts payable……8M
Cash…………………..8M

(k) (k)
Expenses…………………1M Expenses……………14M
Investment in branch………..1M Home office………….1M
Cash…………………..15M

(l) (i)
Investment in branch……3M Expenses………………3M
Expenses……………………….3M Home office……………3M

(m) (m) Adjusting entry:


No entry Inventory – end. ………7.675M
(20.5M x ¼) + (5.1M x ½)
Income summary………7.675M

(o) Adjusting entry: (o) Adjusting entry:


Investment in branch…..135K Depreciation – Bldg. 1M
Accum. Depn. – Equipt…….135K Depreciation – Equpt. 135K
Depreciation – Furn. 75K
Acc. Dep. – Bldg………. 1M
Acc. Dep. – Furn……… 75K
Home office………….. 135K

(p)Closing entries: (p) Closing entries:


Sales……………. 100M
Income summary (m) 7.675M
Shipments from HO…….25M
Freight-in………………. 600K
Purchases………………..10M
Expenses……………… 17M
Investment in branch…..53.865M Depreciation expense….1.21M
Income summary…………53.865M Income summary……53.865M

Income summary……53.865M
Home office…………53.865M

10
Requirement (b):
Investment in branch Home office
Jan. 1 10,000,000 10,000,000 Jan. 1
(a) 30,000,000 900,000 (d) 900,000 30,000,000 (a)
(b) 20,500,000 10,000,000 (h) 10,000,000 20,500,000 (b)
(c) 5,000,000 35,000,000 (i) 35,000,000 5,000,000 (c)
(l) 3,000,000 1,000,000 (k) 1,000,000 3,000,000 (l)
(o) 135,000 135,000 (o)
(p) 53,865,000 53,865,000 (p)
75,600,000 75,600,000

Requirement (c):
Cash 30,400,000
Accounts receivable 10,000,000
Inventory 7,675,000
Land 10,000,000
Building 20,000,000
Accum. Depn. - Bldg. (1,000,000)
Furniture 600,000
Accum. Depn. - Furniture (75,000)
Total assets 77,600,000

Accounts payable 2,000,000


Home office 75,600,000
Total liabilities & equity 77,600,000

Sales 100,000,000
Cost of goods sold:
Shipments from HO 25,000,000
Freight-in 600,000
Purchases 10,000,000
Ending inventory (7,675,000) (27,925,000)
Gross profit 72,075,000
Expenses (17,000,000)
Depreciation expense (1,210,000)
Profit 53,865,000

11
ACTIVITY #2:
Solutions:

Requirement (a):
Home office books Branch books
(a) (a)
Investment in branch……200 Shipments from HO……200
Shipments to the branch……..200 Home office…………… 200

No entry Home office…………… 50


Shipments from HO…… 50

(b) (b)
Investment in branch……... 100 Cash……………… 150
Cash………………………... 100 Home office……………… 150

(c) (c)
No entry Home office……… 20
Cash (or Expense) …… 20

(d) (e)
Investment in branch …………10 No entry
Expense……………………….10

Requirement (b):
Investment in branch Home office
Jan. 1 1,000 1,000 Jan. 1
(a) 200 (a) 50 200 (a)
(b) 100 (c) 20 150 (b)
(d) 10
1,310 1,280

Difference = 30

Requirement (c):
Home office books Branch books
(a) (a)
Shipments to the branch….. 50
Investment in branch……….. 50

(b) (b)
Home office……….. 50
Cash………………………. 50

(c) (c)
Expenses…………….. 20
Investment in branch………..20

12
(d) (d)
Expense 10
Home office………………10

Requirement (d):
Investment in branch Home office
Unadj. 1,310 1,280 Unadj.
50 (a) (b) 50
20 (c) 10 (d)

1,240 1,240

PROBLEM 5: MULTIPLE CHOICE - THEORY


1. B 6. D
2. D 7. D
3. C 8. D
4. D 9. A
5. C 10. A

PROBLEM 11-6: MULTIPLE CHOICE - COMPUTATIONAL


1. B
Solution:
Sales 112,500
Shipments from home office 120,000
Inventory, Dec. 31 (30,000) (90,000)
Gross profit 22,500
Expenses (8,100)
Profit 14,400

2. D
Solution:
Home office Current, unadjusted 90,000
Profit of branch 14,400
Adjusted balance of reciprocal accounts 104,400

3. A (see solutions below)

4. C
Solutions:
(Home office (Branch

13
books) books)
Investment
Home office
in Branch
Unadjusted balances 175,520 184,279.50 squeeze
(a) Charge recorded twice (500)
(b) Mathematical mistake in
805.50
recording (895 – 89.5)
(c) Mathematical mistake in
recording (980-890) 90
(d) Mathematical mistake in
recording (400-350) 50
(e) Unrecorded charge 425
(f) Erroneous credit to investment 5,000
(g) Erroneous debit to HO account 370
(h) Erroneous correcting entry (5,000)
Adjusted balances 180,520 180,520

Notes:
(d) A credit by the home office means a deduction to the “Investment”
account which should have a corresponding deduction also to the
“Home office” account. The deduction of ₱350 was recorded by the
branch as ₱400 resulting to over-deduction. Thus the adjustment is
an addition of ₱50.
(e) The branch failed to record the charge as a credit to the “Home
Office” Account. Instead, branch recorded the charge as a liability.
Thus, the proper adjustment is an increase to the “Home Office”
Account.
(f) No adjustment is needed for the “Home Office” account because
the branch did not take up initially (see ‘h’ below) the erroneous
credit by the home office.
(g) Initially, the branch did not take up the erroneous credit by the
home office in ‘f;’ however, on June 30, 20x1 (cut-off date), the
branch finally recorded the erroneous credit. The proper adjusting
entry is to reverse this. A credit to the “Home Office” account means
an increase; therefore, the correction is a decrease.

5. C
Solution:
Inventory, Dec. 31 28,000
Less: Inventory, Dec. 31 from local purchase (7,000)
Inventory, Dec. 31 from home office at billed price 21,000
Divide by: 140%
Inventory from home office at cost 15,000
Add: Inventory, Dec. 31, from local purchase 7,000
Total ending inventory at cost 22,000

6. D
14
Solution:
Net sales 180,000
Merchandise from home office at cost (98K / 140%) 70,000
Merchandise purchased locally by branch 40,000
Total goods available for sale 110,000
Total ending inventory at cost (22,000) (88,000)
True gross profit 92,000

15
Chapter 12
Insurance Contracts

PROBLEM 1: TRUE OR FALSE


1. FALSE
2. FALSE
3. TRUE
4. FALSE
5. FALSE

PROBLEM 2: FOR CLASSROOM DISCUSSION


1. D
2. D
3. A
4. B
5. D

PROBLEM 3: MULTIPLE CHOICE


1. D
2. C
3. A
4. D
5. D
6. A
7. A
8. B
9. D
10. C
Chapter 13
Accounting for Build-operate-transfer (BOT)

PROBLEM 1: TRUE OR FALSE


1. FALSE 6. TRUE
2. TRUE 7. TRUE
3. FALSE 8. TRUE
4. FALSE 9. FALSE
5. TRUE 10. FALSE

PROBLEM 2: FOR CLASSROOM DISCUSSION


1. D

2. D

3. D

4. B

5. A

6. C
Solution: (15M construction costs x 120%) = 18M

7. C
Solution: (5M operation costs x 105%) = 5.25M

8. B

9. D

10. C
Solution: (15M construction costs x 120%) = 18M

11. D
Solution: 20M, the fees collected from railway users

12. C
Solution:
Total construction costs (15M x 2 yrs.) 30
Multiply by: 120%
Initial carrying amount of intangible asset 36
1
Multiply by: (7 / 8) (a) 7/8
Carrying amount of intangible asset at the end of Year 3 31.50

(a) Denominator in the fraction: The intangible asset’s useful life is 8

years, i.e., Years 3 to 10 where the operator can exercise its right to
collect fees from railway users.

Numerator in the fraction: Amortization starts in Year 3 when the


operator obtains ability to exercise its rights. Thus, at the end of Year
3, the remaining useful life of the intangible asset is 7 years.

PROBLEM 3: EXERCISES
1. A

2. B
Solution:
Int. Amorti-
Date Collections Revenue PV
income zation
b = PV x e = prev.
a c=a-b d
9.10% bal. - c + d
1/1/Yr. 1 -
12/31/Yr.1 - - - 520 520
12/31/Yr.2 - 99 (99) 520 1,139
12/31/Yr.3 300 218 82 11 1,068

The revenues are computed as follows:


Year Performance obligation Revenues
Year 1 Construction services (400M x 130%) 520
Year 2 Construction services (400M x 130%) 520
Year 3 Operation services (10M x 110%) 11

3. C
Solution:
Year 2 Year 3
Revenue 520 11
Contract costs (400) (10)
Interest income 99 218
Profit 219 219

4. C (See solution in preceding question.)

5. B

2
6. D

7. D

8. B
Solution:
Yr. 1 Construction services (400M x 130%) 520
Yr. 1 Borrowing costs (100M x 10%) 10
Yr. 2 Construction services (400M x 130%) 520
Yr. 2 Borrowing costs (100M x 10%) 10
Carrying amt. of intangible asset at the end of Yr. 2 1,060
Amortization expense in Year 3 (132.5)
Carrying amt. of intangible asset at the end of Yr. 3 927.50

The amortization expense is computed as follows (1,060 ÷ 8 = 132.5).

9. B
Solution:
Year 2 Year 3
Revenue 520 300
Contract costs (400) (10)
Amortization expense (132.50) (132.50)
Interest expense - (10)
Profit (Loss) (12.50) 148

10. A (See solution in preceding problem)

PROBLEM 4: THEORY
1. D 6. D
2. B 7. C
3. C 8. B
4. C 9. A
5. E 10. B

You might also like